Формулы по физике по теме кинематика: Формулы, свойства, методы и другая справочная информация

Содержание

Движение с постоянным ускорением и решение задач. Кинематика — 10 класс

Движение с постоянным ускорением и решение задач. Кинематика — 10 класс

Подробности
Просмотров: 739

Кинематика — это просто!

В общем случае движение может быть криволинейным и неравномерным.
Тогда вектор скорости будет меняться и по направлению, и по величине, а это значит, что тело движется с ускорением.
Ускорение показывает быстроту изменения скорости.

Ускорение — это векторная величина, которая характеризуется модулем и направлением.

Единица измерения ускорения в системе СИ:

Частным случаем такого движения является прямолинейное движение с постоянным ускорением.
Постоянное ускорение — это когда ускорение не меняется ни по модулю, ни по направлению.

Прямолинейное движение с постоянным ускорением подразделяется на:
1. равноускоренное, когда при движении модуль скорости тела увеличивается (тело разгоняется).

Здесь векторы скорости и ускорения совпадают по направлению.

2. равнозамедленное, когда при движении модуль скорости тела уменьшается (тело тормозит).
Здесь векторы скорости и ускорения направлены противоположно друг другу.

Формула ускорения:
1. в векторном виде

2. расчетная формула в координатной форме (для решения задач)

Отсюда «вытекает» уравнение скорости, которое выражает мгновенную скорость тела в любой момент времени:
1. в векторном виде

2. расчетная формула в координатной форме

Графики ускорения

Перемещение

1. формула перемещения в векторном виде

2. Расчетная формула в координатной форме

Графики перемещения

Уравнение движения (или иначе уравнение координаты)

1. в векторном виде

2. расчетная формула в координатной форме


Примеры решения задач на движение с постоянным ускорением

Задача 1

Тело движется согласно уравнению х=2-4t-2t2.
Дать описание движения тела.
Составить уравнение скорости движущегося тела.
Определить скорость тела и координату через 10 секунд после начала движения.

Решение

Сравниваем заданное уравнение движения х=2-4t-2t2 с формулой:

тогда

По полученным данным даем описание движения тела:

— тело движется из точки с координатами 2 метра относительно начала координат с начальной скоростью 4 м/с противоположно направлению координатной оси ОХ с постоянным ускорением 4 м/с2, разгоняется, т.к. направление вектора скорости и вектора ускорения совпадают.

Составляем уравнение скорости, глядя на расчетную формулу для скорости:


Расчитываем скорость и координату тела через 10 секунд после начала движения:

Задача 2

Уравнение движения тела x=-3+t+t2
Дать описание движения тела.
Определить скорость и координату тела через 2 секунды после начала движения.

Решение

Рассуждаем аналогично вышерассмотренной задаче:

Тело движется из точки с координатами -3 метра относительно начала координат с начальной скоростью 1 м/с в направлении координатной оси ОХ с постоянным ускорением 2м/с2, разгоняется, т.к. проекции вектора скорости и ускорения имеют одинаковые знаки, значит оба векторв направлены одинаково.



Кинематика — Класс!ная физика

Прямолинейное равномерное движение и решение задач — Закон сложения скоростей и решение задач — Движение с постоянным ускорением и решение задач — Свободное падение — Движение тела, брошенного под углом к горизонту — Решение задач. Тело, брошенное под углом к горизонту — Криволинейное движение

Контрольная работа по теме «Кинематика»

Контрольная работа № 1 «Основы кинематики»

Вариант 1.

  1. Можно ли считать воздушный шарик материальной точкой при определении архимедовой силы, действующей на шар в воздухе?

_________, потому что ______________________________________

  1. Мяч упал с высоты 2 метра и отскочив от земли и был пойман на высоте 1 м. В обоих случаях мяч двигался вдоль вертикальной прямой. Определите путь l пройденный мячом за все время движения и перемещение S при этом движении.

l ___________________, S ___________________.

  1. Д ва автомобиля движутся по прямолинейному участку шоссе. На рисунке изображены графики проекций скоростей на ось х, параллельную шоссе.

    1. Как движутся автомобили: равномерно или равноускорено?

____________________________________________

    1. Как направлены скорости автомобилей по отношению друг к другу?

_____________________________________________

    1. С какими по модулю скоростями движутся автомобили?

_____________________________________________

  1. Скорость скатывающегося с горы лыжника за 3с увеличилась от 0.2 м/с до 2 м/с. Определите проекцию вектора ускорения лыжника на ось х, сонаправленную со скоростью его движения.

_____________________________________________

  1. Поезд движется со скоростью 20 м/с. Чему будет равна скорость поезда после торможения, проходящего с ускорением 0,25 м/с2 через 20 с.

_____________________________________________

  1. На рисунке показано, как меняется с течением времени вектор скорости тела. Пользуясь графиком, определите проекцию ах и модуль а вектора ускорения с которым движется тело.

______________________________________________

  1. Машина, движущаяся со скоростью 15 м/с, тормозит перед пешеходным переходом. Каков тормозной путь машины, если она движется в течении 10с с ускорение 0,5м/с

    2. Запишите решение задачи.

Контрольная работа № 1 «Основы кинематики»

Вариант 2.

  1. Можно ли считать Земной шар материальной точкой при определении времени восхода солнца на восточной и западной границах России.

__________________________________________________________

  1. Средняя точка минутной стрелки часов находиться на расстоянии 2 см от центра циферблата. Определите путь l и перемещение S этой точки за 30 мин, если за час она проходит путь, равный 12,56 см.

l _________________, S _______________

  1. Д ва автомобиля движутся по прямолинейному участку шоссе. На рисунке изображены графики проекций скоростей на ось х, параллельную шоссе.

    1. Как движутся автомобили: равномерно или равноускорено?

__________________________________________

    1. Как направлены скорости автомобилей по отношению друг к другу?

_____________________________________________

    1. С какими по модулю скоростями движутся автомобили?

_____________________________________________

  1. Скатившийся с горы лыжник в течении 6с двигался по равнине. При этом его скорость уменьшилась от 3 м/с до 0. Определите проекцию вектора ускорения на ось х, сонаправленную со скоростью вектора движения лыжника.

__________________________________________

  1. Какую скорость приобретает автомобиль при разгоне с ускорением 0,4 м/с2 в течении 10 с, если начальная скорость движения автомобиля была равна 10 м/с?

__________________________________________

  1. Н а рисунке показано как меняется с течением времени проекция вектора скорости тела. Пользуясь графиком определите проекцию

    ах и модуль а вектора ускорения с которым движется тело. __________________________________________

  1. Какое перемещение совершает самолет за 10с прямолинейного разбега при начальной скорости 10 м/с и ускорении 1,5 м/с2? Запишите решение задачи.

Чат-бот для решения задач по физике по теме «Кинематика»

Актуальность

В самом общем смысле чат-бот – это компьютерная программа, которая интерактивно имитирует человеческую речь (устную или письменную) и позволяет общаться с цифровыми устройствами так, как если бы они были живыми людьми. Чат-боты могут быть очень простыми, как элементарные программы, отвечающие на простой запрос однострочным ответом, или сложными, как цифровые помощники, которые обучаются и развиваются по мере сбора и обработки информации, тем самым повышая свой уровень персонализации.

Большинство школьников сталкиваются с проблемой непонимания задач, например, из курса физики раздела «Механика». Этот бот поможет разобраться с решением задач по теме «Кинематика» и проверить полученные решения в режиме онлайн.

Цель

Разработать обучающую программу для решения задач по физике из раздела «Кинематика», а также создать чат-бот для социальной сети ВКонтакте для повышения доступности и удобства использования программы.

Задачи

  1. Изучить язык программирования Python и проанализировать существующие решения.
  2. Составить алгоритм программы на языке Python, позволяющий решать задачи по физике из темы «Кинематика» (базовый уровень).
  3. Изучить принципы работы VK_API с целью создания чат-бота и дальнейшего его продвижения.
  4. Провести бета-тест чат-бота на уроке физики в классе, получить обратную связь посредством сервиса, интегрированного в чат-бот.

Оснащение и оборудование, использованное при создании работы

  • Персональный компьютер с установленным ПО (pycharm Community Edition 2020.2.3, Python)
  • Смартфон с установленным ПО и выходом в интернет

Описание

Первый этап работы. Большинство школьников сталкиваются с проблемой непонимания некоторых задач по физике. В интернете есть много онлайн-помощников. Но большинство связано лишь с математическими уравнениями/функциями. Всё, что мы нашли в сфере физики, либо было заброшено, либо не работало.

Следующим этапом было выявление возможности разработки программы для решения задач по физике из раздела «Механика», а также создания чат-бота по решению задач из курса физики базового уровня. Далее – визуализация алгоритма работы. Для этого нам не нужны были знания программирования, надо было просто составить блок-схему из последовательных шагов. Алгоритм работы чат-бота удобно представлять в виде mind map. Это инструмент визуализации, который позволяет конвертировать идею в схему из готовых блоков.

Определены формулы для решения задач по кинематике и значения, при  которых уравнения не решаются. Формулы записаны на языке Python, введены переменные, размерности физических величин в системе СИ. Для удобства использования бота сделаны специальные кнопки быстрой клавиатуры, благодаря которым не придётся писать долгий текст о том, что нужно сделать. Достаточно будет нажать на кнопку и получить результат.

Результаты работы/выводы

Автор изучил язык программирования Python, написал базовый алгоритм программы, изучил принципы VK_API, провёл бета-тест чат-бота на уроке физики.

Создан чат-бот, выдающий алгоритм решения задачи, формулы и основные определения из раздела «Механика» по теме кинематика.

Перспективы использования результатов работы

С помощью бота вы сможете подготовиться к самостоятельной или контрольной работе по данному разделу физики. Наш чат-бот содержит полный список формул по разделу «Механика», в его функции входит решение задач с предоставлением формулы из списка формул, с помощью которой производился расчёт, показ расчёта единиц измерений физических величин, перевод в систему СИ.

Сотрудничество с вузом/учреждением при создании работы

Инженерная академия РУДН

Награды/достижения (в каких конкурсах и с какими результатами выставлялась ранее эта работа)

Призер конференции Инженерной академии РУДН

Мнение автора

«Участие в проекте «Инженерный класс в московской школе» помогло представить результаты выполненного проекта на конкурсе проектов и исследований Открытой городской научно-практической конференции «Инженеры будущего». Было замечательно почувствовать себя разработчиком программного контента и научиться представлять результаты работы. Обмен мнениями во время представления проекта поможет его усовершенствовать и представить на следующей конференции!»

Формулы по физике, которые рекомендуется выучить и хорошо освоить для успешной сдачи ЕГЭ.

Физика. Содержание:

Код: Содержание: 1. МЕХАНИКА 1.1. КИНЕМАТИКА 1.1.1. Механическое движение и его виды 1.1.2. Относительность механического движения 1.1.3. Скорость 1.1.4. Ускорение 1.1.5. Равномерное движение 1.1.6. Прямолинейное

Подробнее

Единый государственный экзамен по ФИЗИКЕ

«УТВЕРЖДАЮ» Руководитель Федеральной службы по надзору в сфере образования и науки «СОГЛАСОВАНО» Председатель Научнометодического совета ФИПИ по физике Единый государственный экзамен по ФИЗИКЕ Кодификатор

Подробнее

НАУЧНО-ОБРАЗОВАТЕЛЬНЫЙ МАТЕРИАЛ

НЕКОММЕРЧЕСКАЯ ОРГАНИЗАЦИЯ «АССОЦИАЦИЯ МОСКОВСКИХ ВУЗОВ» ГОСУДАРСТВЕННОЕ ОБРАЗОВАТЕЛЬНОЕ УЧРЕЖДЕНИЕ ВЫСШЕГО ПРОФЕССИОНАЛЬНОГО ОБРАЗОВАНИЯ МОСКОВСКИЙ ГОСУДАРСТВЕННЫЙ УНИВЕРСИТЕТ ГЕОДЕЗИИ И КАРТОГРАФИИ НАУЧНО-ОБРАЗОВАТЕЛЬНЫЙ

Подробнее

2018/2019 учебный год

УТВЕРЖДЕНО Приказ Министра образования Республики Беларусь 03.12.2018 836 Билеты для проведения экзамена в порядке экстерната при освоении содержания образовательной программы среднего образования по учебному

Подробнее

Единый государственный экзамен по ФИЗИКЕ

«УТВЕРЖДАЮ» Директор Федерального института педагогических измерений «СОГЛАСОВАНО» Председатель Научнометодического совета ФИПИ по физике Единый государственный экзамен по ФИЗИКЕ Кодификатор элементов

Подробнее

ПЕНЗЕНСКИЙ ГОСУДАРСТВЕННЫЙ УНИВЕРСИТЕТ

ПЕНЗЕНСКИЙ ГОСУДАРСТВЕННЫЙ УНИВЕРСИТЕТ ПРОГРАММА ВСТУПИТЕЛЬНОГО ЭКЗАМЕНА ПО ФИЗИКЕ Составитель: Профессор, к.т.н. Першенков П.П. Пенза 2014 Механика 1. Прямолинейное равномерное движение. Вектор. Проекции

Подробнее

Пояснительная записка

Пояснительная записка Цель курса: развитие и формирование у учащихся предметных компетенций и их применение в нестандартных задачах. Задачи курса: 1. Углубление содержания курса физики; 2. Развивать самостоятельность

Подробнее

Пояснительная записка

Пояснительная записка Программный материал рассчитан для учащихся 11 классов на 1 учебный час в неделю, всего 34 часа. Настоящая программа позволяет более глубоко и осмысленно изучать практические и теоретические

Подробнее

ПРОГРАММА ВСТУПИТЕЛЬНОГО ИСПЫТАНИЯ

Министерство образования и науки Российской Федерации федеральное государственное автономное образовательное учреждение высшего образования «Санкт-Петербургский политехнический университет Петра Великого»

Подробнее

галактики, Вселенная.

При составлении программы следующие правовые документы 10-11классы были использованы федеральный компонент государственного стандарта среднего (полного) общего образования по физике, утвержденный в 2004

Подробнее

Раздел 1. Планируемые результаты.

Раздел 1. Планируемые результаты. Личностные: в ценностно-ориентированной сфере чувство гордости за российскую физическую науку, отношение к физике как элементу общечеловеческой культуры, гуманизм, положительное

Подробнее

ОСНОВЫ ЭЛЕМЕНТАРНОЙ ФИЗИКИ

Е.Н. Бурцева, В.А. Пивень, Т.Л. Шапошникова, Л.Н. Терновая ОСНОВЫ ЭЛЕМЕНТАРНОЙ ФИЗИКИ (базовый уровень) Учебное пособие Краснодар 2012 УДК 53 ББК 22.3 Б91 Рецензенты: Е.Н. Тумаев, доктор физико-математических

Подробнее

Содержание курса (10 класс)

Планируемые результаты освоения курса Личностные результаты — формирование ответственного отношения к учению, готовности и способности обучающихся к саморазвитию и самообразованию на основе мотивации к

Подробнее

ЗАДАЧИ ПО ФИЗИКЕ С АНАЛИЗОМ ИХ РЕШЕНИЯ

Н.Е.Савченко ЗАДАЧИ ПО ФИЗИКЕ С АНАЛИЗОМ ИХ РЕШЕНИЯ В книге дана методика решения задач но физике с анализом типичных ошибок, допускаемых абитуриентами на вступительных экзаменах. Сборник рекомендуется

Подробнее

РАБОЧАЯ ПРОГРАММА ЭЛЕКТИВНОГО КУРСА

МУНИЦИПАЛЬНОЕ ОБЩЕОБРАЗОВАТЕЛЬНОЕ УЧРЕЖДЕНИЕ «ПРИМОРСКАЯ СРЕДНЯЯ ОБЩЕОБРАЗОВАТЕЛЬНАЯ ШКОЛА» РАБОЧАЯ ПРОГРАММА ЭЛЕКТИВНОГО КУРСА Решение задач различной сложности по физике Категория слушателей: учащиеся

Подробнее

ТРЕБОВАНИЯ К УРОВНЮ ПОДГОТОВКИ УЧАЩИХСЯ

ТРЕБОВАНИЯ К УРОВНЮ ПОДГОТОВКИ УЧАЩИХСЯ В результате изучения физики обучающиеся 7 класса должны смысл понятий: физическое явление, физический закон, вещество. смысл физических величин: путь, скорость,

Подробнее

Формулы по физике основные. Формулы по физике для егэ. Работа, мощность, энергия

Для того чтобы успешно подготовиться к ЦТ по физике и математике, среди прочего, необходимо выполнить три важнейших условия:

  1. Изучить все темы и выполнить все тесты и задания приведенные в учебных материалах на этом сайте. Для этого нужно всего ничего, а именно: посвящать подготовке к ЦТ по физике и математике, изучению теории и решению задач по три-четыре часа каждый день. Дело в том, что ЦТ это экзамен, где мало просто знать физику или математику, нужно еще уметь быстро и без сбоев решать большое количество задач по разным темам и различной сложности. Последнему научиться можно только решив тысячи задач.
  2. Выучить все формулы и законы в физике, и формулы и методы в математике . На самом деле, выполнить это тоже очень просто, необходимых формул по физике всего около 200 штук, а по математике даже чуть меньше. В каждом из этих предметов есть около десятка стандартных методов решения задач базового уровня сложности, которые тоже вполне можно выучить, и таким образом, совершенно на автомате и без затруднений решить в нужный момент большую часть ЦТ. После этого Вам останется подумать только над самыми сложными задачами.
  3. Посетить все три этапа репетиционного тестирования по физике и математике. Каждый РТ можно посещать по два раза, чтобы прорешать оба варианта. Опять же на ЦТ, кроме умения быстро и качественно решать задачи, и знания формул и методов необходимо также уметь правильно спланировать время, распределить силы, а главное правильно заполнить бланк ответов, не перепутав ни номера ответов и задач, ни собственную фамилию. Также в ходе РТ важно привыкнуть к стилю постановки вопросов в задачах, который на ЦТ может показаться неподготовленному человеку очень непривычным.

Успешное, старательное и ответственное выполнение этих трех пунктов, а также ответственная проработка итоговых тренировочных тестов , позволит Вам показать на ЦТ отличный результат, максимальный из того, на что Вы способны.

Нашли ошибку?

Если Вы, как Вам кажется, нашли ошибку в учебных материалах, то напишите, пожалуйста, о ней на электронную почту (). В письме укажите предмет (физика или математика), название либо номер темы или теста, номер задачи, или место в тексте (страницу) где по Вашему мнению есть ошибка. Также опишите в чем заключается предположительная ошибка. Ваше письмо не останется незамеченным, ошибка либо будет исправлена, либо Вам разъяснят почему это не ошибка.

Сессия приближается, и пора нам переходить от теории к практике. На выходных мы сели и подумали о том, что многим студентам было бы неплохо иметь под рукой подборку основных физических формул. Сухие формулы с объяснением: кратко, лаконично, ничего лишнего. Очень полезная штука при решении задач, знаете ли. Да и на экзамене, когда из головы может «выскочить» именно то, что накануне было жесточайше вызубрено, такая подборка сослужит отличную службу.

Больше всего задач обычно задают по трем самым популярным разделам физики. Это механика , термодинамика и молекулярная физика , электричество . Их и возьмем!

Основные формулы по физике динамика, кинематика, статика

Начнем с самого простого. Старое-доброе любимое прямолинейное и равномерное движение.

Формулы кинематики:

Конечно, не будем забывать про движение по кругу, и затем перейдем к динамике и законам Ньютона.

После динамики самое время рассмотреть условия равновесия тел и жидкостей, т.е. статику и гидростатику

Теперь приведем основные формулы по теме «Работа и энергия». Куда же нам без них!


Основные формулы молекулярной физики и термодинамики

Закончим раздел механики формулами по колебаниям и волнам и перейдем к молекулярной физике и термодинамике.

Коэффициент полезного действия, закон Гей-Люссака, уравнение Клапейрона-Менделеева — все эти милые сердцу формулы собраны ниже.

Кстати! Для всех наших читателей сейчас действует скидка 10% на любой вид работы .


Основные формулы по физике: электричество

Пора переходить к электричеству, хоть его и любят меньше термодинамики. Начинаем с электростатики.

И, под барабанную дробь, заканчиваем формулами для закона Ома, электромагнитной индукции и электромагнитных колебаний.

На этом все. Конечно, можно было бы привести еще целую гору формул, но это ни к чему. Когда формул становится слишком много, можно легко запутаться, а там и вовсе расплавить мозг. Надеемся, наша шпаргалка основных формул по физике поможет решать любимые задачи быстрее и эффективнее. А если хотите уточнить что-то или не нашли нужной формулы: спросите у экспертов студенческого сервиса . Наши авторы держат в голове сотни формул и щелкают задачи, как орешки. Обращайтесь, и вскоре любая задача будет вам «по зубам».

Абсолютно необходимы для того, чтобы человек, решивший изучать эту науку, вооружившись ими, мог чувствовать себя в мире физики как рыба в воде. Без знания формул немыслимо решение задач по физике. Но все формулы запомнить практически невозможно и важно знать, особенно для юного ума, где найти ту или иную формулу и когда ее применить.

Расположение физических формул в специализированных учебниках распределяется обычно по соответствующим разделам среди текстовой информации, поэтому их поиск там может отнять довольно-таки много времени, а тем более, если они вдруг понадобятся Вам срочно!

Представленные ниже шпаргалки по физике содержат все основные формулы из курса физики , которые будут полезны учащимся школ и вузов.

Все формулы школьного курса по физике с сайта http://4ege.ru
I. Кинематика скачать
1. Основные понятия
2. Законы сложения скоростей и ускорений
3. Нормальное и тангенциальное ускорения
4. Типы движений
4.1. Равномерное движение
4.1.1. Равномерное прямолинейное движение
4.1.2. Равномерное движение по окружности
4.2. Движение с постоянным ускорением
4.2.1. Равноускоренное движение
4.2.2. Равнозамедленное движение
4.3. Гармоническое движение
II. Динамика скачать
1. Второй закон Ньютона
2. Теорема о движении центра масс
3. Третий закон Ньютона
4. Силы
5. Гравитационная сила
6. Силы, действующие через контакт
III. Законы сохранения. Работа и мощность скачать
1. Импульс материальной точки
2. Импульс системы материальных точек
3. Теорема об изменении импульса материальной точки
4. Теорема об изменении импульса системы материальных точек
5. Закон сохранения импульса
6. Работа силы
7. Мощность
8. Механическая энергия
9. Теорема о механической энергии
10. Закон сохранения механической энергии
11. Диссипативные силы
12. Методы вычисления работы
13. Средняя по времени сила
IV. Статика и гидростатика скачать
1. Условия равновесия
2. Вращающий момент
3. Неустойчивое равновесие, устойчивое равновесие, безразличное равновесие
4. Центр масс, центр тяжести
5. Сила гидростатического давления
6. Давлением жидкости
7. Давление в какой-либо точке жидкости
8, 9. Давление в однородной покоящейся жидкости
10. Архимедова сила
V. Тепловые явления скачать
1. Уравнение Менделеева-Клапейрона
2. Закон Дальтона
3. Основное уравнение МКТ
4. Газовые законы
5. Первый закон термодинамики
6. Адиабатический процесс
7. КПД циклического процесса (теплового двигателя)
8. Насыщенный пар
VI. Электростатика скачать
1. Закон Кулона
2. Принцип суперпозиции
3. Электрическое поле
3.1. Напряженность и потенциал электрического поля, созданного одним точечным зарядом Q
3.2. Напряженность и потенциал электрического поля, созданного системой точечных зарядов Q1, Q2, …
3.3. Напряженность и потенциал электрического поля, созданного равномерно заряженным по поверхности шаром
3.4. Напряженность и потенциал однородного электрического поля, (созданного равномерно заряженной плоскотью или плоским конденсатором)
4. Потенциальная энергия системы электрических зарядов
5. Электроемкость
6. Свойства проводника в электрическом поле
VII. Постоянный ток скачать
1. Упорядоченная скорость
2. Сила тока
3. Плотность тока
4. Закон Ома для участка цепи, не содержащего ЭДС
5. Закон Ома для участка цепи, содержащего ЭДС
6. Закон Ома для полной (замкнутой) цепи
7. Последовательное соединение проводников
8. Параллельное соединение проводников
9. Работа и мощность электрического тока
10. КПД электрической цепи
11. Условие выделения максимальной мощности на нагрузке
12. Закон Фарадея для электролиза
VIII. Магнитные явления скачать
1. Магнитное поле
2. Движение зарядов в магнитном поле
3. Рамка с током в магнитном поле
4. Магнитные поля, создаваемые различными токами
5. Взаимодействие токов
6. Явление электромагнитной индукции
7. Явление самоиндукции
IX. Колебания и волны скачать
1. Колебания, определения
2. Гармонические колебания
3. Простейшие колебательные системы
4. Волна
X. Оптика скачать
1. Закон отражения
2. Закон преломления
3. Линза
4. Изображение
5. Возможные случаи расположения предмета
6. Интерференция
7. Дифракция

Большая шпаргалка по физике . Все формулы изложены в компактном виде с небольшими комментариями. Шпаргалка также содержит полезные константы и прочую информацию. Файл содержит следующие разделы физики:

    Механика (кинематика, динамика и статика)

    Молекулярная физика. Свойства газов и жидкостей

    Термодинамика

    Электрические и электромагнитные явления

    Электродинамика. Постоянный ток

    Электромагнетизм

    Колебания и волны. Оптика. Акустика

    Квантовая физика и теория относительности

Маленькая шпора по физике . Все самое необходимое для экзамена. Нарезка основных формул по физике на одной странице. Не очень эстетично, зато практично. 🙂

Размер: px

Начинать показ со страницы:

Транскрипт

1 Формулы по физике, которые рекомендуется выучить и хорошо освоить для успешной сдачи ЕГЭ. Версия: 0.92 β. Составитель: Ваулин Д.Н. Литература: 1. Пёрышкин А.В. Физика 7 класс. Учебник для общеобразовательных учреждений. 13-е издание, стереотипное. Москва. Дрофа Пёрышкин А.В. Физика 8 класс. Учебник для общеобразовательных учреждений. 12-е издание, стереотипное. Москва. Дрофа Пёрышкин А.В., Гутник Е.М. Физика 9 класс. Учебник для общеобразовательных учреждений. 14-е издание, стереотипное. Москва. Дрофа Мякишев Г.Я. и др. Физика. Механика 10 класс. Профильный уровень. Учебник для общеобразовательных учреждений. 11-е издание, стереотипное. Москва. Дрофа Мякишев Г.Я., Синяков А.З. Физика. Молекулярная физика. Термодинамика 10 класс. Профильный уровень. Учебник для общеобразовательных учреждений. 13-е издание, стереотипное. Москва. Дрофа Мякишев Г.Я., Синяков А.З., Слободсков Б.А. Физика. Электродинамика классы. Профильный уровень. Учебник для общеобразовательных учреждений. 11-е издание, стереотипное. Москва. Дрофа Мякишев Г.Я., Синяков А.З. Физика. Колебания и волны 11 класс. Профильный уровень. Учебник для общеобразовательных учреждений. 9-е издание, стереотипное. Москва. Дрофа Мякишев Г.Я., Синяков А.З. Физика. Оптика. Квантовая физика 11 класс. Профильный уровень. Учебник для общеобразовательных учреждений. 9-е издание, стереотипное. Москва. Дрофа Жирным выделены формулы, которые стоит учить, когда уже отлично освоены не выделенные жирным формулы. 7 класс. 1. Средняя скорость: 2. Плотность: 3. Закон Гука: 4. Сила тяжести:

2 5. Давление: 6. Давление столба жидкости: 7. Архимедова сила: 8. Механическая работа: 9. Мощность совершения работы: 10. Момент силы: 11. Коэффициент полезного действия (КПД) механизма: 12. Потенциальная энергия при постоянном: 13. Кинетическая энергия: 8 класс. 14. Количество теплоты необходимое для нагревания: 15. Количество теплоты, выделяемое при сгорании: 16. Количество теплоты необходимое для плавления:

3 17. Относительная влажность воздуха: 18. Количество теплоты необходимое для парообразования: 19. КПД теплового двигателя: 20. Полезная работа теплового двигателя: 21. Закон сохранения заряда: 22. Сила тока: 23. Напряжение: 24. Сопротивление: 25. Общее сопротивление последовательного соединения проводников: 26. Общее сопротивление параллельного соединения проводников: 27. Закон Ома для участка цепи:

4 28. Мощность электрического тока: 29. Закон Джоуля-Ленца: 30. Закон отражения света: 31. Закон преломления света: 32. Оптическая сила линзы: 9 класс. 33. Зависимость скорости от времени при равноускоренном движении: 34. Зависимость радиус вектора от времени при равноускоренном движении: 35. Второй закон Ньютона: 36. Третий закон Ньютона: 37. Закон всемирного тяготения:

5 38. Центростремительное ускорение: 39. Импульс: 40. Закон изменения энергии: 41. Связь периода и частоты: 42. Связь длинны волны и частоты: 43. Закон изменения импульса: 44. Закон Ампера: 45. Энергия магнитного поля тока: 46. Формула трансформатора: 47. Действующее значение тока: 48. Действующее значение напряжения:

6 49. Заряд конденсатора: 50. Электроёмкость плоского конденсатора: 51. Общая ёмкость параллельно соединённых конденсаторов: 52. Энергия электрического поля конденсатора: 53. Формула Томпсона: 54. Энергия фотона: 55. Поглощение фотона атомом: 56. Связь массы и энергии: 1. Поглощённая доза излучения: 2. Эквивалентная доза излучения:

7 57. Закон радиоактивного распада: 10 класс. 58. Угловая скорость: 59. Связь скорости с угловой: 60. Закон сложения скоростей: 61. Сила трения скольжения: 62. Сила трения покоя: 3. Сила сопротивления среды: [ 63. Потенциальная энергия растянутой пружины: 4. Радиус вектор центра масс:

8 64. Количество вещества: 65. Уравнение Менделеева-Клапейрона: 66. Основное уравнение молекулярно кинетической теории: 67. Концентрация частиц: 68. Связь между средней кинетической энергией частиц и температурой газа: 69. Внутренняя энергия газа: 70. Работа газа: 71. Первое начало термодинамики: 72. КПД машины Карно: 5. Тепловое линейное расширение: 6. Тепловое объёмное расширение:

9 73. Закон Кулона: 74. Напряжённость электрического поля: 75. Напряжённость электрического поля точечного заряда: 7. Поток напряжённости электрического поля: 8. Теорема Гаусса: 76. Потенциальная энергия заряда при постоянном: 77. Потенциальная энергия взаимодействия тел: 78. Потенциальная энергия взаимодействия зарядов: 79. Потенциал: 80. Разность потенциалов: 81. Связь напряжённости однородного электрического поля и напряжения:

10 82. Общая электроёмкость последовательно соединённых конденсаторов: 83. Зависимость удельного сопротивления от температуры: 84. Первое правило Кирхгофа: 85. Закон Ома для полной цепи: 86. Второе правило Кирхгофа: 87. Закон Фарадея: 11 класс. 9. Закон Био-Савара-Лапласа: 10. Магнитная индукция бесконечного провода: 88. Сила Лоренца:

11 89. Магнитный поток: 90. Закон электромагнитной индукции: 91. Индуктивность: 92. Зависимость величины, изменяющейся по гармоническому закону от времени: 93. Зависимость скорости изменения величины, изменяющейся по гармоническому закону от времени: 94. Зависимость ускорения изменения величины, изменяющейся по гармоническому закону от времени: 95. Период колебаний нитяного маятника: 96. Период колебаний пружинного маятника: 11. Емкостное сопротивление: 12. Индуктивное сопротивление:

12 13. Сопротивление для переменного тока: 97. Формула тонкой линзы: 98. Условие интерференционного максимума: 99. Условие интерференционного минимума: 14. Преобразования Лоренца координат: 15. Преобразования Лоренца времени: 16. Релятивистский закон сложения скоростей: 100. Зависимость массы тела от скорости: 17. Релятивистская связь между энергией и импульсом:

13 101. Уравнение фотоэффекта: 102. Красная граница фотоэффекта: 103. Длина волны Де Бройля:


Программа вступительных испытаний по учебному предмету «Физика» для лиц, имеющих общее среднее образование, для получения высшего образования І ступени, 2018 год 1 УТВЕРЖДЕНО Приказ Министра образования

ФЕДЕРАЛЬНОЕ ГОСУДАРСТВЕННОЕ БЮДЖЕТНОЕ ОБРАЗОВАТЕЛЬНОЕ УЧРЕЖДЕНИЕ ВЫСШЕГО ОБРАЗОВАНИЯ «АНГАРСКИЙ ГОСУДАРСТВЕННЫЙ ТЕХНИЧЕСКИЙ УНИВЕРСИТЕТ» УТВЕРЖДАЮ «чебной работе II.В. Истомина 2016 г. ПРОГРАММА ВСТУПИТЕЛЬНОГО

2 6. Количество заданий в одном варианте теста 30. Часть А 18 заданий. Часть В 12 заданий. 7. Структура теста Раздел 1. Механика 11 заданий (36,7 %). Раздел 2. Основы молекулярно-кинетической теории и

УТВЕРЖДЕНО Приказ Министра образования Республики Беларусь от 30.10.2015 817 Программы вступительных испытаний в учреждения образования для лиц, имеющих общее среднее образование, для получения высшего

1/5 ПРОГРАММА ВСТУПИТЕЛЬНЫХ ИСПЫТАНИЙ ФИЗИКА 1. МЕХАНИКА КИНЕМАТИКА Механическое движение и его виды. Относительность механического движения. Скорость. Ускорение. Равномерное движение. Прямолинейное равноускоренное

1. Общие положения Программа предназначена для подготовки к вступительному испытанию по физике для поступающих на факультет физики и ИКТ Чеченского государственного университета. Вступительный экзамен

Код: Содержание: 1. МЕХАНИКА 1.1. КИНЕМАТИКА 1.1.1. Механическое движение и его виды 1.1.2. Относительность механического движения 1.1.3. Скорость 1.1.4. Ускорение 1.1.5. Равномерное движение 1.1.6. Прямолинейное

ПРОГРАММА ЭЛЕМЕНТОВ СОДЕРЖАНИЯ И ТРЕБОВАНИЙ К УРОВНЮ ПОДГОТОВКИ ВЫПУСКНИКОВ ОБЩЕОБРАЗОВАТЕЛЬНЫХ УЧРЕЖДЕНИЙ ДЛЯ ПРОВЕДЕНИЯ В 2014 ГОДУ ВСТУПИТЕЛЬНЫХ ИСПЫТАНИЙ ПО ФИЗИКЕ Программа элементов содержания по

ПРОГРАММА СОБЕСЕДОВАНИЯ ПО ДИСЦИПЛИНЕ «ФИЗИКА» Физика и методы научного познания Предмет физики. Физика как наука. Научные методы познания окружающего мира и их отличия от других методов познания. Физика

СПЕЦИФИКАЦИЯ теста по учебному предмету «Физика» для проведения централизованного тестирования в 2017 году 1. Назначение теста объективное оценивание уровня подготовки лиц, имеющих общее среднее образование

СПЕЦИФИКАЦИЯ теста по учебному предмету «Физика» для проведения централизованного тестирования в 2018 году 1. Назначение теста объективное оценивание уровня подготовки лиц, имеющих общее среднее образование

Оглавление Основные положения… 3 1. МЕХАНИКА… 3 2. МОЛЕКУЛЯРНАЯ ФИЗИКА. ТЕПЛОВЫЕ ЯВЛЕНИЯ… 4 3. ОСНОВЫ ЭЛЕКТРОДИНАМИКИ… 4 4. КОЛЕБАНИЯ И ВОЛНЫ… 5 5. ОПТИКА… 5 6. КВАНТОВАЯ ФИЗИКА… 6 СПИСОК

1 Общие положения Настоящая программа составлена на основе действующих учебных программ для средней школы, колледжа и техникума. При проведении собеседования основное внимание обращается на понимание абитуриентами

Спецификация теста по предмету физика для Единого национального тестирования и комплексного тестирования (Утвержден для использования в Едином национальном тестировании и комплексном тестировании с 2018

ПРОГРАММА ВСТУПИТЕЛЬНЫХ ИСПЫТАНИЙ (БАКАЛАВРИАТ/СПЕЦИАЛИТЕТ) ПО ОБЩЕОБРАЗОВАТЕЛЬНОЙ ДИСЦИПЛИНЕ «ФИЗИКА» Программа составлена на основе Федерального государственного образовательного стандарта среднего общего

«УТВЕРЖДАЮ» Руководитель Федеральной службы по надзору в сфере образования и науки «СОГЛАСОВАНО» Председатель Научнометодического совета ФИПИ по физике Единый государственный экзамен по ФИЗИКЕ Кодификатор

По предмету: Физика, 11 класс 2017 г. СОДЕРЖАНИЕ 1. Перечень диагностических работ 2. Количественные показатели 3. Общие результаты 3.1. Результаты на уровне региона 3.2. Распределение по баллам 3.3. Результаты

НЕКОММЕРЧЕСКАЯ ОРГАНИЗАЦИЯ «АССОЦИАЦИЯ МОСКОВСКИХ ВУЗОВ» ГОСУДАРСТВЕННОЕ ОБРАЗОВАТЕЛЬНОЕ УЧРЕЖДЕНИЕ ВЫСШЕГО ПРОФЕССИОНАЛЬНОГО ОБРАЗОВАНИЯ МОСКОВСКИЙ ГОСУДАРСТВЕННЫЙ УНИВЕРСИТЕТ ГЕОДЕЗИИ И КАРТОГРАФИИ НАУЧНО-ОБРАЗОВАТЕЛЬНЫЙ

УТВЕРЖДЕНО Приказ Министра образования Республики Беларусь 03.12.2018 836 Билеты для проведения экзамена в порядке экстерната при освоении содержания образовательной программы среднего образования по учебному

ПРОГРАММА ВСТУПИТЕЛЬНЫХ ЭКЗАМЕНОВ ПО ФИЗИКЕ В первом столбце указан код раздела, которому соответствуют крупные блоки содержания. Во втором столбце приводится код элемента содержания, для которого создаются

ПРОГРАММА ВСТУПИТЕЛЬНЫХ ИСПЫТАНИЙ ПО ФИЗИКЕ САНКТ-ПЕТЕРБУРГ 2014 ГОД 1. Механическое движение. Относительность движения. Системы отсчета. Материальная точка. 2. Траектория. Путь и перемещение. 3. Равномерное

Министерство образования и науки Краснодарского края государственное бюджетное профессиональное образовательное учреждение Краснодарского края «Краснодарский информационно- технологический техникум» Тематический

Подготовка к ЕГЭ по физике (4 месяца) Перечень лекций, тестов и заданий. Дата начала Дата завершения Блок 0 Введение В.1 Скалярные и векторные величины. В.2 Сложение и вычитание векторов. В.3 Умножение

Введение………………………………. 8 Руководство по использованию диска…………….. 8 Установка программы……………………. 8 Работа с программой……………………. 11 От издательства…………………………

Негосударственное образовательное учреждение высшего образования «Кубанский социально-экономический институт (КСЭИ)» ПРОГРАММА ВСТУПИТЕЛЬНЫХ ИСПЫТАНИЙ ПО ФИЗИКЕ для абитуриентов, поступающих в вуз Рассмотрено

ПРОГРАММА ВСТУПИТЕЛЬНОГО ИСПЫТАНИЯ ПО ФИЗИКЕ В ФГБОУ ВО «ПГУ» В 2016 ГОДУ СОДЕРЖАНИЕ ПРОГРАММЫ 1 МЕХАНИКА 1.1 КИНЕМАТИКА 1.1.1 Механическое движение и его виды 1.1.2 Относительность механического движения

ПРОГРАММА ВСТУПИТЕЛЬНОГО ИСПЫТАНИЯ ПО ФИЗИКЕ для поступающих в Московский государственный университет геодезии и картографии. Программа составлена в соответствии с типовой программой по физике средней

Министерство образования и науки Российской Федерации Федеральное государственное бюджетное образовательное учреждение высшего профессионального образования «Московский государственный строительный университет»

Вопросы к экзаменационным билетам по дисциплине Физика Билет 1 1. Физика и метод научного познания. Современная физическая картина мира. 2. Магнитное поле. Магнитное взаимодействие. Вектор магнитной индукции.

«УТВЕРЖДАЮ» Директор Федерального института педагогических измерений «СОГЛАСОВАНО» Председатель Научнометодического совета ФИПИ по физике Единый государственный экзамен по ФИЗИКЕ Кодификатор элементов

Тематика тестовых задач по физике для 11 класса Механика Кинематика: 1. Кинематика прямолинейного движения материальной точки. Путь и перемещение. Скорость и ускорение. Сложение скоростей. Прямолинейное

ÓÄÊ 373:53 ÁÁÊ 22.3ÿ72 Í34 Макет подготовлен при содействии ООО «Айдиономикс» В оформлении обложки использованы элементы дизайна: Tantoon Studio, incomible / Istockphoto / Thinkstock / Fotobank.ru Í34

ПЕНЗЕНСКИЙ ГОСУДАРСТВЕННЫЙ УНИВЕРСИТЕТ ПРОГРАММА ВСТУПИТЕЛЬНОГО ЭКЗАМЕНА ПО ФИЗИКЕ Составитель: Профессор, к.т.н. Першенков П.П. Пенза 2014 Механика 1. Прямолинейное равномерное движение. Вектор. Проекции

МИНИСТЕРСТВО ОБОРОНЫ РОССИЙСКОЙ ФЕДЕРАЦИИ Федеральное государственное казённое военное образовательное учреждение высшего образования Краснодарское высшее военное авиационное училище лётчиков имени Героя

189 УТВЕРЖДЕНО Приказ Министра образования Республики Беларусь от 30.10.2018 765 Программа вступительных испытаний по учебному предмету «Физика» для лиц, имеющих общее среднее образование, для получения

Программа вступительных испытаний по учебному предмету «Физика» для лиц, имеющих общее среднее образование, для получения высшего образования І ступени или среднего специального образования, 2019 год ПОЯСНИТЕЛЬНАЯ

Контрольные работы по физике 29 группа 4 семестр Решаем один из предложенных вариантов в каждой контрольной работе. Контрольная работа 11 Механические колебания. Упругие волны. Вариант 1 1. Материальная

Программа к вступительному испытанию по общеобразовательному предмету «Физика» при поступлении в Сыктывкарский лесной институт Программа предназначена для подготовки к массовой письменной проверке знаний

Федеральное государственное автономное учреждение высшего профессионального образования Национальный исследовательский университет «Высшая школа экономики» Программа вступительного испытания по физике

Пояснительная записка Программный материал рассчитан для учащихся 11 классов на 1 учебный час в неделю, всего 34 часа. Настоящая программа позволяет более глубоко и осмысленно изучать практические и теоретические

ФГБОУ ВПО «Петербургский государственный университет путей сообщения Императора Александра I» Программа вступительного испытания по физике для поступающих на обучение по программам бакалавриата и специалитета

ПРОГРАММА ВСТУПИТЕЛЬНОГО ЭКЗАМЕНА ПО ФИЗИКЕ для абитуриентов, поступающих в ФГБОУ ВО Смоленскую ГСХА в 2017 году Программа для вступительного испытания по физике Раздел 1. Перечень элементов содержания,

Занят ия Наименование разделов и дисциплин 1 Механическое движение. Относительность механического движения. Система отсчёта. Материальная точка. Траектория. Путь. Вектор перемещения и его проекции. Прямолинейное

Аннотация к рабочей программе по физике 7 класс (базовый уровень) Рабочая программа по физике 7 класса составлена на основании ФЗ РФ 273 от компонента государственного стандарта основного общего образования

1 семестр Введение. 1 Основные науки о природе. Естественнонаучный метод познания. Раздел 1. Механика. Тема 1.1. Кинематика твёрдого тела 2 Относительность механического движения. Системы отсчета. Характеристики

2 ификатор элементов содержания и требований к уровню подготовки выпускников общеобразовательных учреждений для проведения единого государственного экзамена по ФИЗИКЕ Единый государственный экзамен по

ПРОГРАММА ПО ФИЗИКЕ При проведении экзаменов по физике основное внимание должно быть обращено на понимание экзаменующимся сущности физический явлений и законов, на умение истолковать смысл физических величин

Программа по физике для поступающих в ОАНО ВПО ВУиТ Вступительные испытания по физике проводятся в форме письменной работы (тестирования) и собеседования, с помощью которой проверяются знания учащихся,

Министерство образования и науки Российской Федерации федеральное государственное автономное образовательное учреждение высшего образования «Санкт-Петербургский политехнический университет Петра Великого»

ЭКЗАМЕНАЦИОННЫЕ БИЛЕТЫ ГОСУДАРСТВЕННОЙ ИТОГОВОЙ АТТЕСТАЦИИ ПО ФИЗИКЕ ПО ОБРАЗОВАТЕЛЬНЫМ ПРОГРАММАМ ОСНОВНОГО ОБЩЕГО ОБРАЗОВАНИЯ Билет 1 1. Что изучает физика. Физические явления. Наблюдения, опыты. 2.

МИНИСТЕРСТВО ОБРАЗОВАНИЯ РЕСПУБЛИКИ БЕЛАРУСЬ Учреждение образования «Брестский государственный технический университет» ПРОГРАММА собеседования для иностранных абитуриентов по предмету «ФИЗИКА» Разработана:

Аннотация к рабочим программам по физике Класс: 10 Уровень изучения учебного материала: базовый. УМК, учебник: Рабочая программа по физике для 10-11 классов составлена на основе Федерального компонента

Методы научного познания Эксперимент и теория в процессе познания мира. Моделирование явлений. Физические законы и пределы их применения. Роль математики в физике. Принципы причинности и соответствия.

ФЕДЕРАЛЬНОЕ АГЕНТСТВО ЖЕЛЕЗНОДОРОЖНОГО ТРАНСПОРТА федеральное государственное бюджетное образовательное учреждение высшего профессионального образования «ОМСКИЙ ГОСУДАРСТВЕННЫЙ УНИВЕРСИТЕТ ПУТЕЙ СООБЩЕНИЯ»

Аннотация к контрольно-оценочному средству по учебному предмету «Физика» 1. Общие положения. Контрольно-оценочные средства (КОС) предназначены для контроля и оценки образовательных достижений обучающихся,

При составлении программы следующие правовые документы 10-11классы были использованы федеральный компонент государственного стандарта среднего (полного) общего образования по физике, утвержденный в 2004

Раздел 1. Планируемые результаты. Личностные: в ценностно-ориентированной сфере чувство гордости за российскую физическую науку, отношение к физике как элементу общечеловеческой культуры, гуманизм, положительное

Е.Н. Бурцева, В.А. Пивень, Т.Л. Шапошникова, Л.Н. Терновая ОСНОВЫ ЭЛЕМЕНТАРНОЙ ФИЗИКИ (базовый уровень) Учебное пособие Краснодар 2012 УДК 53 ББК 22.3 Б91 Рецензенты: Е.Н. Тумаев, доктор физико-математических

0 Пояснительная записка. Программа по физике для 10 11 классов составлена на основе авторской программы: Физика 10 11 класс Г.Я. Мякишев М.:Дрофа,-2010г. и ориентирована на использование учебно-методического

Тема Дата Количество часов Календарно-тематическое планирование По физике 10 класс (профильный уровень) Требования к знаниям Форма контроля ФИЗИКА И МЕТОДЫ НАУЧНОГО ПОЗНАНИЯ 1 ФИЗИЧЕСКИЕ ЗАКОНЫ И ТЕОРИИ

Как правило, именно математику, а не физику принято считать королевой точных наук. Мы полагаем, что это утверждение спорно, ведь технический прогресс невозможен без знания физики и её развития. Из-за своей сложности она вряд ли когда-либо будет включена в список обязательных государственных экзаменов, но, так или иначе, абитуриентам технических специальностей приходится сдавать её в обязательном порядке. Труднее всего запомнить многочисленные законы и формулы по физике для ЕГЭ, именно о них мы расскажем в этой статье.

Секреты подготовки

Возможно, это связано с кажущейся сложностью предмета или популярностью профессий гуманитарного и управленческого профиля, но в 2016 году только 24 % всех абитуриентов приняли решение сдавать физику, в 2017 — лишь 16 %. Такие статистические данные невольно заставляют задуматься, не слишком ли завышены требования или просто уровень интеллекта в стране падает. Почему-то не верится, что так мало школьников 11 класса желают стать:

  • инженерами;
  • ювелирами;
  • авиаконструкторами;
  • геологами;
  • пиротехниками;
  • экологами,
  • технологами на производстве и т.д.

Знание формул и законов физики в равной степени необходимо для разработчиков интеллектуальных систем, вычислительной техники, оборудования и вооружения. При этом всё взаимосвязано. Так, например, специалисты, производящие медицинское оборудование, в своё время изучали углубленный курс атомной физики, ведь без разделения изотопов, у нас не будет ни рентгенологической аппаратуры, ни лучевой терапии. Поэтому создатели ЕГЭ постарались учесть все темы школьного курса и, кажется, не пропустили ни одной.

Те ученики, которые исправно посещали все уроки физики вплоть до последнего звонка, знают, что в период с 5 по 11 класс изучается около 450 формул. Выделить из этих четырех с половиной сотен хотя бы 50 крайне сложно, поскольку все они важны. Подобного мнения, очевидно, также придерживаются разработчики Кодификатора. Тем не менее, если вы одарены необыкновенно и не ограничены во времени, вам хватит 19 формул, ведь при желании из них можно вывести все остальные. За основу мы решили взять главные разделы:

  • механику;
  • физику молекулярную;
  • электромагнетизм и электричество;
  • оптику;
  • физику атомную.

Очевидно, что подготовка к ЕГЭ должна быть ежедневной, но если по каким-то причинам вы приступили к изучению всего материала лишь сейчас, настоящее чудо может совершить экспресс-курс, предлагаемый нашим центром. Надеемся, эти 19 формул также будут вам полезны:

Вы, наверное, заметили, что некоторые формулы по физике для сдачи ЕГЭ остались без пояснений? Мы предоставляем вам самим их изучить и открыть для себя законы, по которым абсолютно всё вершится в этом мире.

Краткое содержание школьного курса физики Механика Формулы кинематики Кинематика Механическое движение

Краткое содержание


школьного курса физики

Механика

Формулы кинематики:

Кинематика

Механическое движение

Механическим движением называется изменение положения тела (в пространстве) относительно других тел (с течением времени).

Относительность движения. Система отсчета

Чтобы описать механическое движение тела (точки), нужно знать его координаты в любой момент времени. Для определения координат следует выбрать ­тело отсчета и связать с ним систему координат. Часто телом отсчета служит Земля, с которой связывается прямоугольная декартова система координат. Для определения положения точки в любой момент времени необходимо также задать начало отсчета времени.

Система координат, тело отсчета, с которым она связана, и прибор для измерения времени образуют систему отсчета, относительно которой рассматривается движение тела.

Материальная точка

Тело, размерами которого в данных условиях движения можно пренебречь, называют материальной точкой.

Тело можно рассматривать как материальную точку, если его размеры малы по сравнению с расстоянием, которое оно проходит, или по сравнению с расстояниями от него до других тел.

Траектория, путь, перемещение

Траекторией движения называется линия, вдоль которой движется тело. Длина траектории называется пройденным путем. Путь – скалярная физическая величина, может быть только положительным.

Перемещением называется вектор, соединяющий начальную и конечную точки траектории.

Движение тела, при котором все его точки в данный момент времени движутся одинаково, называется поступательным движением. Для описания поступательного движения тела достаточно выбрать одну точку и описать ее движение.

Движение, при котором траектории всех точек тела являются окружностями с центрами на одной прямой и все плоскости окружностей перпендикулярны этой прямой, называется вращательным движением.

Метр и секунда

Чтобы определить координаты тела, необходимо уметь измерять расстояние на прямой между двумя точками. Любой процесс измерения физической величины заключается в сравнении измеряемой величины с единицей измерения этой величины.

Единицей измерения длины в Международной системе единиц (СИ) является метр. Метр равен примерно 1/40 000 000 части земного меридиана. По современному представлению метр – это расстояние, которое свет проходит в пустоте за 1/299 792 458 долю секунды.

Для измерения времени выбирается какой-нибудь периодически повторяющийся процесс. Единицей измерения времени в СИ принята секунда. Секунда равна 9 192 631 770 периодам излучения атома цезия при переходе между двумя уровнями сверхтонкой структуры основного состояния.

В СИ длина и время приняты за независимые от других величины. Подобные величины называются основными.

Мгновенная скорость

Для количественной характеристики процесса движения тела вводится понятие скорости движения.

Мгновенной скоростью поступательного движения тела в момент времени t называется отношение очень малого перемещения s к малому промежутку времени t, за который произошло это перемещение:

; .

Мгновенная скорость – векторная величина. Мгновенная скорость перемещения всегда направлена по касательной к траектории в сторону движения тела.

Единицей скорости является 1 м/с. Метр в секунду равен скорости прямолинейно и равномерно движущейся точки, при которой точка за время 1 с перемещается на расстояние 1 м.

Ускорение

Ускорением называется векторная физическая величина, равная отношению очень малого изменения вектора скорости к малому промежутку времени, за которое произошло это изменение, т.е. это мера быстроты изменения скорости:

; .

Метр в секунду за секунду – это такое ускорение, при котором скорость тела, движущегося прямолинейно и равноускоренно, за время 1 с изменяется на 1 м/с.

Направление вектора ускорения совпадает с направлением вектора изменения скорости () при очень малых значениях промежутка времени, за который происходит изменение скорости.

Если тело движется по прямой и его скорость возрастает, то направл­ение вектора ускорения совпадает с направлением вектора скорости; при убывании скорости – противоположно направлению вектора скорости.

При движении по криволинейной траектории направление вектора скорости изменяется в процессе движения, вектор ускорения при этом может оказаться направлен под любым углом к вектору скорости.

Равномерное, равноускоренное прямолинейное движение

Движение с постоянной скоростью называется равномерным прямолинейным движением. При равномерном прямолинейном движении тело движется по прямой и за любые равные промежутки времени проходит одинаковые пути.

Движение, при котором тело за равные промежутки времени совершает неодинаковые перемещения, называют неравномерным движением. При таком движении скорость тела изменяется с течением времени.

Равнопеременным называется такое движение, при котором скорость тела за любые равные промежутки времени изменяется на одинаковую величину, т.е. движение с постоянным ускорением.

Равноускоренным называется равнопеременное движение, при котором величина скорости возрастает. Равнозамедленным – равнопеременное движение, при котором величина скорости уменьшается.

Сложение скоростей

Рассмотрим перемещение тела в подвижной системе координат. Пусть  – перемещение тела в подвижной системе координат,  – перемещение подвижной системы координат относительно неподвижной, тогда  – перемещение тела в неподвижной системе координат равно:

.

Если перемещения и совершаются одновременно, то:

.

Таким образом

.

Мы получили, что скорость тела относительно неподвижной системы отсчета равна сумме скорости тела в подвижной системе отсчета и скорости подвижной системы отсчета относительно неподвижной. Это утверждение называется классическим законом сложения скоростей.

Графики зависимости кинематических величин от времени

в равномерном и равноускоренном движении

При равномерном движении:
  1. График скорости – прямая y = b;

  2. График ускорения – прямая y = 0;

  3. График перемещения – прямая y = kx+b.

При равноускоренном движении:
  1. График скорости – прямая y = kx+b;

  2. График ускорения – прямая y = b;

  3. График перемещения – парабола:

    • если a>0, ветви вверх;

    • чем больше ускорение, тем уже ветви;

    • вершина совпадает по времени с моментом, когда скорость тела равна нулю;

    • как правило, проходит через начало отсчета.

Свободное падение тел. Ускорение свободного падения

Свободным падением называется такое движение тела, когда на него действует только сила тяжести.

При свободном падении ускорение тела направлено вертикально вниз и примерно равно 9,8 м/с2. Это ускорение называется ускорением свободного падения и одинаково для всех тел.

Равномерное движение по окружности

При равномерном движении по окружности значение скорости постоянно, а ее направление изменяется в процессе движения. Мгновенная скорость тела всегда направлена по касательной к траектории движения.

Т.к. направление скорости при равномерном движении по окружности постоянно изменяется, то это движение всегда равноускоренное.

Промежуток времени, за который тело совершает полный оборот при движении по окружности, называется периодом:

.

Т.к. длина окружности s равна 2R, период обращения при равномерном движении тела со скоростью v по окружности радиусом R равен:

.

Величина, обратная периоду обращения, называется частотой обращения и показывает, сколько оборотов по окружности совершает тело в единицу времени:

.

Угловой скоростью называется отношение угла, на который повернулось тело, к времени поворота:

.

Угловая скорость численно равна числу оборотов за 2 секунд.

Ускорение при равномерном движении тел по окружности (центростремительное ускорение)

При равномерном движении по окружности тело движется с центростремительным ускорением. Определим это ускорение.

Ускорение направлено туда же, куда и изменение скорости, следовательно, ускорение направлено к центру окружности. Важное допущение: угол  настолько мал, что длина хорды AB совпадает с длиной дуги:

.

по двум пропорциональным сторонам и углу между ними. Следовательно:

– модуль центростремительного ускорения.

Основы динамики

Первый закон Ньютона. Инерциальные системы отсчета.

Принцип относительности Галилея

Любое тело остается неподвижным, пока на него не действуют другие тела. Тело, двигавшееся с некоторой скоростью, продолжает двигаться равномерно и прямолинейно до тех пор, пока не него не подействуют другие тела. К таким выводам о законах движения тел впервые пришел итальянский ученый Галилео Галилей.

Явление сохранения скорости движения тела при отсутствии внешних воздействий называется инерцией.

Всякий покой и движение тел относительны. Одно и то же тело может находиться в состоянии покоя в одной системе отсчета и двигаться с ускорением в другой. Но существуют такие системы отсчета, относительно которых поступательно движущиеся тела сохраняют свою скорость постоянной, если на них не действуют другие тела. Это утверждение называется первым законом Ньютона (законом инерции).

Системы отсчета, относительно которых тело при отсутствии внешних воздействий движется прямолинейно и равномерно, называют инерциальными системами отсчета.

Инерциальных систем отсчета может быть сколь угодно много, т.е. любая система отсчета, которая движется равномерно и прямолинейно по отношению к инерциальной, также является инерциальной. Истинных (абсолютных) инерциальных систем отсчета нет.

Масса

Причиной изменения скорости движения тел всегда является его взаимодействие с другими телами.

При взаимодействии двух тел всегда изменяются скорости и первого, и второго тела, т.е. оба тела приобретают ускорения. Ускорения двух взаимодействующих тел могут быть различными, они зависят от инертности тел.

Инертность – способность тела сохранять свое состояние движения (покоя). Чем больше инертность тела, тем меньшее ускорение оно приобретет при взаимодействии с другими телами, и тем будет ближе его движение к равномерному прямолинейному движению по инерции.

Масса – физическая величина, характеризующая инертность тела. Чем большей массой обладает тело, тем меньшее ускорение оно получает при взаимодействии.

За единицу массы в СИ принят килограмм: [m]=1 кг.

Сила

В инерциальных системах отсчета любое изменение скорости тела происходит под действием других тел. Сила – это количественное выражение действия одного тела на другое.

Сила – векторная физическая величина, за ее направление принимают направление ускорения тела, которое вызывается этой силой. У силы всегда есть точка приложения.

В СИ за единицу силы принимаются сила, которая телу массой 1 кг сообщает ускорение 1 м/с2. Эта единица называется Ньютоном:

.

Второй закон Ньютона

Сила, действующая на тело, равна произведению массы тела на сообщаемое этой силой ускорение:

.

Таким образом, ускорение тела прямо пропорционально действующей на тело силе и обратно пропорционально его массе:

.

Сложение сил

При одновременном действии на одно тело нескольких сил тело движется с ускорением, являющимся векторной суммой ускорений, которые бы возникли под действием каждой силы в отдельности. Действующие на тело силы, приложенные к одной точке, складываются по правилу сложения векторов.

Векторная сумма всех сил, одновременно действующих на тело, называется равнодействующей силой.

Прямая, проходящая через вектор силы, называется линией действия силы. Если силы приложены к разным точкам тела и действуют не параллельно друг другу, то равнодействующая приложена к точке пересечения линий действия сил. Если силы действуют параллельно друг другу, то точки приложения результирующей силы нет, а линия ее действия определяется формулой: (см. рисунок).

Момент силы. Условие равновесия рычага

Основным признаком взаимодействия тел в динамике является возникновение ускорений. Однако часто бывает нужно знать, при каких условиях тело, на которое действует несколько различных сил, находится в состоянии равновесия.

Существует два вида механического движения – поступательное движение и вращение.

Если траектории движения всех точек тела одинаковы, то движение поступательное. Если траектории всех точек тела – дуги концентрических окружностей (окружностей с одним центром – точкой вращения), то движение вращательное.

Равновесие невращающихся тел: невращающееся тело находится в равновесии, если геометрическая сумма сил, приложенных к телу, равна нулю.

Равновесие тела, имеющего неподвижную ось вращения

Если линия действия силы, приложенной к телу, проходит через ось вращения тела, то эта сила уравновешивается силой упругости со стороны оси вращения.

Если линия действия силы не пересекает ось вращения, то эта сила не может быть уравновешена силой упругости со стороны оси вращения, и тело поворачивается вокруг оси.

Вращение тела вокруг оси под действием одной силы может быть остановлено действием второй силы. Опыт показывает, что если две силы по отдельности вызывают вращение тела в противоположных направлениях, то при их одновременном действии тело находится в равновесии, если выполняется условие:

,
где d1 и d2 – кратчайшие расстояния от линий действия сил F1 и F2. Расстояние d называется плечом силы, а произведение модуля силы на плечо – моментом силы:

.

Если моментам сил, вызывающим вращение тела вокруг оси по часовой стрелке, приписать положительный знак, а моментам сил, вызывающим вращение против часовой стрелки, – отрицательный знак, то условие равновесия тела, имеющего ось вращения, можно сформулировать в виде правила моментов: тело, имеющее неподвижную ось вращения, находится в равновесии, если алгебраическая сумма моментов всех приложенных к телу сил относительно этой оси равна нулю:

.

За единицу вращающего момента в СИ принимается момент силы в 1 Н, линия действия которой находится на расстоянии 1 м от оси вращения. Эту единицу называют ньютон-метром.

Общее условие равновесия тела: тело находится в равновесии, если равны нулю геометрическая сумма всех приложенных к нему сил и алгебраическая сумма моментов этих сил относительно оси вращения.

При выполнении этого условия тело необязательно находится в покое. Оно может двигаться равномерно и прямолинейно или вращаться.

Виды равновесия

Равновесие называют устойчивым, если после небольших внешних воздействий тело возвращается в исходное состояние равновесия. Это происходит, если при небольшом смещении тела в любом направлении от первоначального положения равнодействующая сил, действующих на тело, становится отличной от нуля и направлена к положению равновесия.

Равновесие называется неустойчивым, если при небольшом смещении тела из положения равновесия равнодействующая приложенных к нему сил отлична от нуля и направлена от положения равновесия.

Равновесия называется безразличным, если при небольших смещениях тела из первоначального положения равнодействующая приложенных к телу сил остается равной нулю.

Центр тяжести

Центром тяжести называется точка, через которую проходит равнодействующая сил тяжести при любом расположении тела.

Третий закон Ньютона

Тела действуют друг на друга с силами, вдоль одной прямой, равными по модулю и противоположными по направлению. Эти силы имеют одинаковую физическую природу; они приложены к разным телам и поэтому друг друга не компенсируют.

Сила упругости. Закон Гука

Сила упругости возникает в результате деформации тела и направлена в сторону, противоположную деформации.

При малых по сравнению с размерами тел деформациях сила упругости прямо пропорциональна величине абсолютной деформации тела. В проекции на направление деформирования сила упругости равна

,
где x – абсолютная деформация, k – коэффициент жесткости.

Этот закон был установлен экспериментально английским ученым Робертом Гуком и называется законом Гука:

Сила упругости, возникающая при деформации тела, пропорциональна удлинению тела и направлена в сторону, противоположную направлению перемещений частиц тела при деформации.

Коэффициент пропорциональности в законе Гука называется жесткостью тела. Он зависит от формы и размеров тела и от материала, из которого оно изготовлено (уменьшается с увеличением длины и с уменьшением площади поперечного сечения – см. Молекулярную Физику).

В Си жесткость выражается в ньютонах на метр: .

Упругая сила стремится восстановить форму тела, подвергнутого деформации, и приложена к телу, которое эту деформацию вызывает.

Природа силы упругости электромагнитная, т.к. сила упругости возникает в результате стремления электромагнитных сил, действующих между атомами вещества, вернуть атомы вещества в исходное положение при изменении их взаимного положения в результате деформации.

Упругая реакция опоры, нити, подвеса – пассивная сила, действующая всегда перпендикулярно поверхности опоры.

Сила трения. Коэффициент трения скольжения

Сила трения возникает при соприкосновении поверхностей двух тел и всегда препятствует их взаимному перемещению.

Сила, возникающая на границе соприкосновения тел при отсутствии относительного движения называется силой трения покоя. Сила трения покоя – упругая сила, она равна по модуля внешней силе, направленной по касательной к поверхности соприкосновения тел, и противоположна ей по направлению.

При движении одного тела по поверхности другого возникает сила трения скольжения.

Сила трения имеет электромагнитную природу, т.к. возникает благодаря существованию сил взаимодействия между молекулами и атомами соприкасающихся тел – электромагнитных сил.

Сила трения скольжения прямо пропорциональна силе нормального давления (или упругой реакции опоры) и не зависит от площади поверхности соприкосновения тел {закон Кулона}:

, где  – коэффициент трения.

Коэффициент трения зависит от рельефа поверхности и всегда меньше единицы: «сдвинуть легче, чем оторвать».

Гравитационные силы. Закон всемирного тяготения.

Сила тяжести

Согласно законам Ньютона, движение тела с ускорением возможно только под действием силы. Т.к. падающие тела движутся с ускорением, направленным вниз, то на них действует сила притяжения к Земле. Но не только Земля обладает свойством действовать на все тела силой притяжения. Исаак Ньютон предположил, что между всеми телами действуют силы притяжения. Эти силы называются силами всемирного тяготения или гравитационными силами.

Распространив установленные закономерности – зависимость силы притяжения тел к Земле от расстояний между телами и от масс взаимодействующих тел, полученные в результате наблюдений,– Ньютон открыл в 1682 г. закон всемирного тяготения: Все тела притягиваются друг к другу, сила всемирного тяготения прямо пропорциональна произведению масс тел и обратно пропорциональна квадрату расстояния между ними:

.

Векторы сил всемирного тяготения направлены вдоль прямой, соединяющей тела. Коэффициент пропорциональности G называется гравитационной постоянной (постоянной всемирного тяготения) и равна

.

Силой тяжести называется сила притяжения, действующая со стороны Земли на все тела:

.

Пусть – масса Земли, а – радиус Земли. Рассмотрим зависимость ускорения свободного падения от высоты подъема над поверхностью Земли:

Вес тела. Невесомость

Вес тела – сила, с которой тело давит на опору или подвес вследствие притяжения этого тела к земле. Вес тела приложен к опоре (подвесу). Величина веса тела зависит от того, как движется тело с опорой (подвесом).

Вес тела, т.е. сила, с которой тело действует на опору, и сила упругости, с которой опора действует на тело, в соответствие с третьим законом Ньютона равны по абсолютному значению и противоположны по направлению.

Если тело находится в покое на горизонтальной опоре или равномерно движется, на него действуют только сила тяжести и сила упругости со стороны опоры, следовательно вес тела равен силе тяжести (но эти силы приложены к разным телам):

.

При ускоренном движении вес тела не будет равен силе тяжести. Рассмотрим движение тела массой m под действием сил тяжести и упругости с ускорением. По 2-му закону Ньютона:

Если ускорение тела направлено вниз, то вес тела меньше силы тяжести; если ускорение тела направлено вверх, то все тела больше силы тяжести.

Увеличение веса тела, вызванное ускоренным движением опоры или подвеса, называют перегрузкой.

Если тело свободно падает, то из формулы * следует, что вес тела равен нулю. Исчезновение веса при движении опоры с ускорением свободного падения называется невесомостью.

Состояние невесомости наблюдается в самолете или космическом корабле при движении их с ускорением свободного падения независимо от скорости их движения. За пределами земной атмосферы при выключении реактивных двигателей на космический корабль действует только сила всемирного тяготения. Под действием этой силы космический корабль и все тела, находящиеся в нем, движутся с одинаковым ускорением; поэтому в корабле наблюдается явление невесомости.

Движение тела под действием сил тяжести. Движение искусственных спутников. Первая космическая скорость

Если модуль перемещения тела много меньше расстояния до центра Земли, то можно считать силу всемирного тяготения во время движения постоянной, а движение тела равноускоренным. Самый простой случай движения тела под действием силы тяжести – свободное падение с нулевой начальной скоростью. В этом случае тело движется с ускорением свободного падения к центру Земли. Если есть начальная скорость, направленная не по вертикали, то тело движется по криволинейной траектории (параболе, если не учитывать сопротивление воздуха).

При некоторой начальной скорости тело, брошенное по касательной к поверхности Земли, под действием силы тяжести при отсутствии атмосферы может двигаться по окружности вокруг Земли, не падая на нее и не удаляясь от нее. Такая скорость называется первой космической скоростью, а тело, движущееся таким образом – искусственным спутником Земли (ИСЗ).

Определим первую космическую скорость для Земли. Если тело под действием силы тяжести движется вокруг Земли равномерно по окружности, то ускорение свободного падения является его центростремительным ускорением:

.

Отсюда первая космическая скорость равна

.

Первая космическая скорость для любого небесного тела определяется таким же образом. Ускорение свободного падения на расстоянии R от центра небесного тела можно найти, воспользовавшись вторым законом Ньютона и законом всемирного тяготения:

.

Следовательно, первая космическая скорость на расстоянии R от центра небесного тела массой M равна

.

Для запуска на околоземную орбиту ИСЗ необходимо сначала вывести за пределы атмосферы. Поэтому космические корабли стартуют вертикально. На высоте 200 – 300 км от поверхности Земли, где атмосфера разрежена и почти не влияет на движение ИСЗ, ракета делает поворот и сообщает ИСЗ первую космическую скорость в направлении, перпендикулярном вертикали.

Законы сохранения в механике

Импульс тела

По 2-му закону Ньютона изменение скорости тела возможно только в результате его взаимодействия с другими телами, т.е. при действии силы. Пусть на тело массой m в течение времени t действует сила F и скорость его движения изменяется от vo до v. Тогда на основании 2-го закона Ньютона:

.

Величина называется импульсом силы. Импульс силы – это векторная физическая величина, равная произведению силы на время ее действия. Направление импульса силы совпадает с направлением силы.

.

импульс тела (количество движения) – векторная физическая величина, равная произведению массы тела на его скорость. Направление импульса тела совпадает с направлением скорости.

Импульс силы, действующей на тело, равен изменению импульса тела.

Закон сохранения импульса

Выясним, как изменяются импульсы двух тел при их взаимодействии. Обозначим скорости тел массами m1 и m2 до взаимодействия через и , а после взаимодействия – через и .

По 3-му закону Ньютона силы, действующие на тела при их взаимодействии, равны по модулю и противоположны по направлению; поэтому из можно обозначить F и –F. Тогда:

.

Таким образом, векторная сумма импульсов двух тел до взаимодействия равна векторной сумме их импульсов после взаимодействия.

Эксперименты показывают, что в любой системе взаимодействующих между собой тел при отсутствии действия сил со стороны других тел, не входящих в систему, – в замкнутой системе – геометрическая сумма импульсов тел остается постоянной. Импульс замкнутой системы тел есть величина постоянная – закон сохранения импульса (з.с.и.).

Реактивное движение

В реактивном двигателе при сгорании топлива образуются газы, нагретые до высокой температуры, которые выбрасываются из сопла двигателя. Двигатель и выбрасываемые им газы взаимодействуют между собой. На основании з.с.и. при отсутствии внешних сил сумма векторов импульсов взаимодействующих тел остается постоянной. До начала работы двигателя импульс двигателя и горючего был равен нулю, следовательно, после включения двигателя сумма векторов импульса ракеты и импульса истекающих газов равна нулю:

.

Эта формула применима для вычисления скорости двигателя при условии небольшого изменения его массы в результате сгорания топлива.

Реактивный двигатель обладает замечательным свойством: для движения ему не нужны ни земля, ни вода, ни воздух, т.к. он двигается в результате взаимодействия с газами, образующимися при сгорании топлива. Поэтому реактивный двигатель может двигаться в безвоздушном космическом пространстве.

Механическая работа

Механическая работа – это скалярная физическая величина, равная произведению модуля силы на модуль перемещения точки приложения силы и на косинус угла между направлением действия силы и направления перемещения (скалярное произведение векторов силы и точки ее перемещения):

.

Работа измеряется в Джоулях. 1 Джоуль – работа, которую совершает сила 1 Н при перемещении точки ее приложения на 1 м в направлении действия силы:

.

Работа может быть положительной, отрицательной, равной нулю:

  1.  = 0  A = FS > 0;

  2. 0 <  < 90  A > 0;

  3.  = 90  A = 0;

  4. 90 <  < 180 A < 0;

  5.  = 180  A = –FS < 0.

Сила, действующая перпендикулярно перемещению, работы не совершает.

Мощность

Мощность – это работа, совершаемая в единицу времени:

– средняя мощность.

. 1 Ватт – это мощность, при которой совершается работа 1 Дж за 1 с.

Мгновенная мощность:

.

Кинетическая энергия

Установим связь между работой постоянной силы и изменением скорости тела. Рассмотрим случай, когда на тело действует постоянная сила и направление действия силы совпадает с направлением перемещения тела:

. *

Физическая величина, равная половине произведения массы тела на его скорость называется кинетической энергией тела:

.

Тогда из формулы *: – теорема о кинетической энергии: Изменение кинетической энергии тела равно работе всех сил, действующих на тело.

Кинетическая энергия всегда положительна, т.е. зависит от выбора системы отсчета.

Вывод: в физике абсолютное значение энергии вообще, и кинетической энергии в частности, смысла не имеет. Речь может идти только о разнице энергий или об изменении энергии.

Энергия – способность тела совершать работу. Работа – мера изменения энергии.

Потенциальная энергия

Потенциальная энергия – это энергия взаимодействия тел, зависит от взаимного их расположения.

Работа силы тяжести (потенциальная энергия тела в поле силы тяжести)

Если тело перемещается вверх, работа силы тяжести отрицательна; вниз – положительна.

Работа силы тяжести не зависит от траектории движения тела, а зависит лишь от перепада высот (от изменения положения тела над поверхностью земли).

Работа силы тяжести по замкнутому контуру равна нулю.

Силы, работа которых по замкнутому контуру равна нулю, называются потенциальными (консервативными). В механике потенциальными являются сила тяжести и упругая сила (в электродинамике – сила Кулона), непотенциальными – сила трения (в электродинамике – сила Ампера, Лоренца).

Потенциальная энергия тела в поле силы тяжести: .

Работа потенциальной силы всегда равна убыли потенциальной энергии:

.

Работа упругой силы (потенциальная энергия упруго деформированного тела)

/* Если какая-то физическая величина изменяется по линейному закону, ее среднее значение равно полусумме начального и конечного значений – Fy */

Потенциальная энергия упруго деформированного тела: .

Закон сохранения полной механической энергии

Полная механическая энергия – сумма кинетической и потенциальной энергии всех тел, входящих в систему:

.

По теореме о кинетической энергии работа всех сил, действующих на все тела . Если в системе все силы потенциальные, то справедливо утверждение: . Следовательно:

Полная механическая энергия замкнутой системы есть величина постоянная (если в системе действуют только потенциальные силы).

Если в системе есть силы трения, то можно применить следующий прием: силу трения назначаем внешней силой и применяем закон изменения полной механической энергии:

.

Работа внешней силы равна изменению полной механической энергии системы.

Жидкости и газы

Давление

Давление – это физическая величина, численно равная силе нормального давления, действующей на единицу площади:

.

Сила нормального давления всегда действует перпендикулярно поверхности.

.

1 Паскаль – это такое давление, которое производит сила 1 Н на перпендикулярную к ней поверхность площадь 1 м2. На практике применяют и внесистемные единицы давления:

Закон Паскаля для жидкостей и газов

Давление, оказываемое на жидкость, передается ей по всем направлениям одинаково. Давление не зависит от направления.

Гидростатическим давлением называется вес столба жидкости, приходящегося на единицу площади:

.

Такое давление жидкость оказывает на дно и стенки сосуда на глубине h.

Сообщающиеся сосуды

Равенство давлений жидкости на одной и той же высоте приводит к тому, что в сообщающихся сосудах любой формы свободные поверхности покоящейся однородной жидкости находятся на одном уровне (если влияние капиллярных сил пренебрежимо мало).

Если в сообщающиеся сосуды налиты жидкости с различной плотностью, то при равенстве давлений высота столба жидкости с меньшей плотностью будет больше высоты столба жидкости с большей плотностью, т.к. на одной высоте давление одинаково.

Принцип устройства гидравлического пресса

Основными частями гидравлического пресса являются два цилиндра с поршнями. Под цилиндрами находится мало сжимаемая жидкость, цилиндры соединены трубкой, по которой может перетекать жидкость.

При действии силы F1 на поршень в узком цилиндре создается некоторое давление. По закону Паскаля такое же давление создается внутри жидкости во втором цилиндре, т.е.

.

Гидравлический пресс дает выигрыш во столько раз, во сколько раз площадь его большего поршня больше площади малого поршня.

Гидравлический пресс используется в домкратах и тормозных системах.

Атмосферное давление. Изменение атмосферного давления

с высотой

Под действием силы тяжести верхние слои воздуха в земной атмосфере давят на нижележащие слои. Это давление согласно закону Паскаля передается по всем направлениям. Наибольшее значение это давление, называемое атмосферным, имеет у поверхности Земли.

В ртутном барометре вес ртутного столбика, приходящийся на единицу площади (гидростатическое давление ртути), уравновешивается весом столба атмосферного воздуха, приходящегося на единицу площади – атмосферным давлением (см. рисунок).

С увеличение высоты над уровнем моря атмосферное давление уменьшается (см. график).

Архимедова сила для жидкостей и газов. Условия плавания тел

На тело, погруженное в жидкость или в газ, действует выталкивающая сила, направленная вертикально вверх и равная весу жидкости (газа), взятому в объеме погруженного тела.

Формулировка Архимеда: тело теряет в жидкости в весе ровно столько, сколько весит вытесненная их жидкость.

.

.

Вытесняющая сила приложена в геометрическом центре тела (для однородных тел – в центре тяжести).

На тело, находящееся в жидкости или газе, в обычных земных условиях действуют две силы: силы тяжести и архимедова сила. Если сила тяжести по модулю больше архимедовой силы, то тело тонет.

Если модуль силы тяжести равен модулю архимедовой силы, то тело может находиться в равновесии на любой глубине.

Если архимедова сила по модулю больше силы тяжести, то тело всплывает. Всплывшее тело частично выступает над поверхностью жидкости; объем погруженной части тела таков, что вес вытесненной жидкости равен весу плавающего тела.

Архимедова сила больше силы тяжести, если плотность жидкости больше плотности погруженного тела, и наоборот.

Все формулы по физике 10 класс тема механика :: nacomgaces

 

 

 

 

 

 

 

 

 

 

 

 

 

 

 

 

 

 

 

 

Материалы по теме: Кинематика формулы. Кинематика формулы. Основные формулы по физикеМЕХАНИКА. Занимательная физика. Опубликовано :13Князев Олег Анатольевич. Механика, кинематика. Движение по окружности закон всемирного тяготения закон Гука сила трения сила и импульс закон сохранения импульса. Здесь далеко не все формулы. Князев Олег Анатольевич. Решение задач по физике, подготовка к ЭГЕ и ГИА, механика термодинамика.

Мои выпускники. Класс. Основные формулы. Кинематика. Основные формулы кинематики. Подготовка к ЕГЭ, ГИА для школьников 511 классов. Образа модели и справедливы все представленные ниже определения и формулы. И не только механики. Тестызадачи по атомной и ядерной физике длякласса. Скачать формулы по механике в виде файла. Формула. Обозначения. Формулы.

По физике для ЕГЭ и 7 11 класса. И не только может понадобиться 7, 8, 9, и 11 классам. Механика. Тестызадачи.класса. Перейти к списку задач и тестов по теме. Физикаодна из самых удивительных наук. Бесплатные уроки, тесты и тренажёры по физике за класс по. Кинематика. Механика. Основные формулы механики, молекулярной физики,.

Электродинамики. Кинематика. Профильный уровень. Комментарий. Кинематика. Новости науки. Молекулярная физика. Основные формулы школьного курса механики. В дальнейшем, при изучении других разделов физики, Вы неоднократно в этом убедитесь. Мне нравится. Учебник для общеобразовательных учреждений.11 е издание, стереотипное. Москва. Дрофа 2009. Вы можете скачать данные формулы все в одном архиве. Механикаосновные формулы для решения задач. Формулы по физике. На заметку. Дополнительные.

Кинематику, динамику и статику. Основные формулы. Кинематика. Движение, при котором все точки тела движутся одинаково, называется. Поделиться. Формулы по разделамтемам: МЕХАНИКАКинематика прямолинейного движения, кинематика криволинейного. Законы сохранения в механике. Формулировки физических законов и правил из курса 9 класса. Лицей интернат естественных наук 2011 г.2. Механика. Формулы механики. Физика. Механика класс.

И др. В разделе кинематика рассматриваются такие кинематические характеристики движения, как перемещение, скорость, ускорение. Если вам понравился сайт, предлагаем разместить нашу кнопку. Электричество. Оптика. Онлайн калькуляторы. Таблицы. Механика Основные формулы.1. Кинематика.1.1 Скорость тела. В Википедии имеется статья по теме. Описывают только некоторый класс материальных систем,. Механика делится на три раздела:.

 

Вместе с Все формулы по физике 10 класс тема механика часто ищут

 

механика физика формулы и определения.

физика 10 класс формулы кинематика.

формулы динамики.

механика формулы 1 курс.

формулы по физике 10 класс с пояснениями.

физика формулы 10 класс.

механика формулы 9 класс.

кинематика формулы 9 класс

 

Читайте также:

 

Готовые ответы в тетради по геометрии 7 класс атанасян 2012 год

 

Гдз ридер 4 класс

 

Скачать бесплатно без регистрации тесты по физике чеботарёва 8 класс

 

1.4: Кинематика — Физика LibreTexts

Уравнения движения

Ладно, хватит определений. Давайте посмотрим, как все это сочетается друг с другом и как их можно использовать. То, что мы будем рассматривать, называется уравнениями движения , и эта тема часто упоминается как кинематика . Важно отметить, что мы пока не имеем дело с причинами этих движений, а только с самими движениями.

В основном мы будем иметь дело только с постоянными ускорениями (если не указано иное), и поскольку мгновенное ускорение является производной скорости, в случае постоянного ускорения нетрудно интегрировать его, чтобы получить мгновенную скорость как функцию времени:

\ [\ left.\ begin {array} {l} a = \ dfrac {dv} {dt} \; \; \; \ Rightarrow \; \; \; v \ left (t \ right) = \ int {a \; dt} = at + const \; \\ const = v \ left ({t = 0} \ right) \ Equiv { v_o} \ end {array} \ right \} \; \; \; v \ left (t \ right) = at + {v_o} \]

Константа интегрирования находится путем подстановки \ (t = 0 \) в уравнение 1.4.1, что приводит к скорости объекта в начальный момент времени, который обычно обозначается как \ (v_o \).

Мы можем сыграть точно в ту же игру, чтобы получить уравнение движения для положения как функции времени, поскольку мы знаем, как оно соотносится с мгновенной скоростью:

\ [\ left.2} + {v_o} t + {x_o} \]

Обратите внимание: если у нас есть все подробности этого последнего уравнения, мы можем получить уравнение скорости, указанное выше, просто взяв производную. Мы не можем пойти в обратном направлении, не получив также стартовую позицию.

Пример \ (\ PageIndex {1} \)

Ускорение частицы, движущейся вдоль оси x, задается уравнением [обратите внимание, что это , а не постоянная !] :

\ [a \ left (t \ right) = \ beta + \ lambda t, \; \; \; \; \; \; \ beta = 2.3} \ nonumber \]

Частица находится в позиции \ (x = + 4.60m \) и движется в направлении \ (- x \) со скоростью \ (12.0 \ dfrac {m} {s} \) в момент \ (t = 0 с \).

  1. Найдите время, в которое частица (ненадолго) останавливается.
  2. Найдите положение, в котором частица (ненадолго) останавливается.
Решение

а. Начнем с нахождения уравнения для скорости, так как нас интересует время, в которое это значение обращается в ноль.2 + v_o \ cdot 0 + const \; \; \; \Правая стрелка \;\;\; const = 4,60 м \ nonumber \]

И, наконец, вставьте этот результат обратно в уравнение для \ (x \ left (t \ right) \) и вставьте время, которое мы нашли выше (\ (t = 4s \)), чтобы получить:

\ [x \ left (t = 4 \ right) = \ boxed {-21.0m} \ nonumber \]

Давайте подсчитаем все числа, с которыми мы можем столкнуться в ситуации постоянного ускорения:

  • независимая переменная: \ (t \)
  • зависимых переменных: \ (x \), \ (v \)
  • постоянные движения: \ (x_o \), \ (v_o \), \ (a \) (ускорение по предположению постоянное)

Имея шесть чисел для работы, вы можете себе представить, что существует множество способов создать проблему для решения чего-то неизвестного. 2} + {v_o} t} {t} = \ frac {1} {2} at + v_o = \ frac {1} {2} \ left (v_f — v_o \ right) + v_o \; \; \; \Правая стрелка \;\;\; v_ {ave} = \ dfrac {v_o + v_f} {2} \]

Для постоянного ускорения средняя скорость просто равна среднему арифметическому начальной и конечной скоростей.Мы лучше поймем, почему это так получается, когда мы вскоре начнем обсуждать построение графиков.

Свободное падение

Есть один тип движения по прямой с постоянным ускорением, с которым все мы знакомы: свободное падение.

Мы более подробно рассмотрим, как объяснить это в терминах сил в следующем разделе, но если предположить, что сопротивление воздуха оказывает небольшое влияние (помните, мы разрабатываем здесь упрощенную модель), то оказывается (как показано Галилео, отбрасывающим камни из Пизанской башни, и в демонстрации более драматично), что все объекты ускоряются с той же постоянной скоростью, что и падают на Землю.2} \)

Обратите внимание, что единицы измерения расстояния в квадрат времени — это единицы ускорения. Это ускорение, конечно, всегда направлено вниз, и в зависимости от нашего выбора системы координат оно может быть как положительным, так и отрицательным. После выбора системы координат знак для \ (g \) остается неизменным независимо от того, в какую сторону движется объект. Если выбрано положительное направление вверх, а объект движется вверх, то его скорость положительна, а отрицательное значение \ (g \) приводит к замедлению движения объекта.Если он движется вниз, то его скорость отрицательна, а отрицательное ускорение приводит к тому, что скорость становится более отрицательной (т. Е. Ускоряется).

Пример \ (\ PageIndex {2} \)

Мяч бросается вертикально вверх в тот же момент, когда второй мяч падает с места, находясь прямо над ним. Два шара находятся на расстоянии \ (12,0 м \) друг от друга, когда начинают движение. Найдите максимальную скорость, с которой может быть брошен первый шар, чтобы он не столкнулся со вторым шаром, прежде чем он вернется на исходную высоту. 2} + v_o \ left (\ sqrt {\ frac {2y_o} {g}} \ right) = -y_o + \ sqrt {\ frac {2y_o} {g}} v_o \ nonumber \]

Мы настаиваем на том, чтобы брошенный шар упал, по крайней мере, до своей начальной высоты к тому моменту, когда упавший шар достигнет этого уровня, поэтому мы хотим, чтобы \ (y_1 \) не было больше нуля, что дает нам неравенство для \ ( в_о \):

\ [v_o \ le \ sqrt {\ dfrac {g y_o} {2}} \; \; \; \Правая стрелка \;\;\; в_о \ ле 7.67 \ frac {m} {s} \ nonumber \]

Уравнения кинематики — SAT II Physics

Если вы считаете, что контент, доступный через Веб-сайт (как определено в наших Условиях обслуживания), нарушает или несколько ваших авторских прав, сообщите нам, отправив письменное уведомление («Уведомление о нарушении»), содержащее в информацию, описанную ниже, назначенному ниже агенту. Если репетиторы университета предпримут действия в ответ на ан Уведомление о нарушении, оно предпримет добросовестную попытку связаться со стороной, которая предоставила такой контент средствами самого последнего адреса электронной почты, если таковой имеется, предоставленного такой стороной Varsity Tutors.

Ваше Уведомление о нарушении прав может быть отправлено стороне, предоставившей доступ к контенту, или третьим лицам, таким как в качестве ChillingEffects.org.

Обратите внимание, что вы будете нести ответственность за ущерб (включая расходы и гонорары адвокатам), если вы существенно искажать информацию о том, что продукт или действие нарушает ваши авторские права. Таким образом, если вы не уверены, что контент находится на Веб-сайте или по ссылке с него нарушает ваши авторские права, вам следует сначала обратиться к юристу.

Чтобы отправить уведомление, выполните следующие действия:

Вы должны включить следующее:

Физическая или электронная подпись правообладателя или лица, уполномоченного действовать от их имени; Идентификация авторских прав, которые, как утверждается, были нарушены; Описание характера и точного местонахождения контента, который, по вашему мнению, нарушает ваши авторские права, в \ достаточно подробностей, чтобы позволить репетиторам университетских школ найти и точно идентифицировать этот контент; например нам требуется а ссылка на конкретный вопрос (а не только на название вопроса), который содержит содержание и описание к какой конкретной части вопроса — изображению, ссылке, тексту и т. д. — относится ваша жалоба; Ваше имя, адрес, номер телефона и адрес электронной почты; а также Ваше заявление: (а) вы добросовестно считаете, что использование контента, который, по вашему мнению, нарушает ваши авторские права не разрешены законом, владельцем авторских прав или его агентом; (б) что все информация, содержащаяся в вашем Уведомлении о нарушении, является точной, и (c) под страхом наказания за лжесвидетельство, что вы либо владелец авторских прав, либо лицо, уполномоченное действовать от их имени.

Отправьте жалобу нашему уполномоченному агенту по адресу:

Чарльз Кон Varsity Tutors LLC
101 S. Hanley Rd, Suite 300
St. Louis, MO 63105

Или заполните форму ниже:

Тема 2: Механика — IB Physics

См. Руководство по этой теме.

2.1 — Движение
  • Расстояние и перемещение

Расстояние Рабочий объем
Скаляр Вектор
Скалярная величина, которая измеряет расстояние между двумя местоположениями на определенном пути. Векторная величина, определяемая длиной и направлением отрезка линии, соединяющего начальную и конечную позиции объекта.

Скорость Скорость
Скаляр Вектор
Скорость изменения расстояния во времени. Скорость изменения смещения во времени.

Скорость — это мера, зависящая от движения наблюдателя.2.

График вытеснения-времени

Наклонный градиент указывает скорость.

Прямые линии подразумевают постоянную скорость.

График скорости-времени

Наклонный градиент указывает на ускорение.

Прямые наклонные линии подразумевают постоянное ускорение или замедление.

Область под линиями показывает изменение смещения.

График времени разгона

Горизонтальные линии подразумевают постоянное ускорение.

Область под линиями показывает изменение скорости.

  • Уравнения движения для равномерного ускорения

s = смещение u = начальная скорость v = конечная скорость a = ускорение t = затраченное время

Если ускорение постоянное (равномерное), можно использовать следующие уравнения

Считается, что объект подвергается движению снаряда, когда он следует по кривой траектории из-за влияния силы тяжести.2

  • Снаряд достигает максимальной высоты, когда его вертикальная скорость равна нулю
  • Траектория симметричная
  • Наличие сопротивления воздуха изменяет траекторию полета снаряда на следующие

    • Максимальная высота снаряда меньше
    • Дальность полета снаряда короче
    • Траектория несимметричная
    • Сопротивление жидкости и конечная скорость

    Сопротивление воздуха ограничивает максимальную скорость, которую объект может получить при свободном падении.Например:

    • Если вы выпрыгнете из самолета и испытаете свободное падение, вы почувствуете восходящую силу, действующую на вас со стороны окружающего воздуха из-за сопротивления воздуха.
    • По мере того, как вы падаете все быстрее и быстрее из-за силы тяжести, эта восходящая сила, создаваемая воздухом, становится все больше и больше, пока она не уравновесит ваш вес. В этот момент результирующая сила, действующая на вас, становится равной нулю, и вы больше не ускоряетесь.
    • Эта конкретная скорость, с которой вы прекращаете ускоряться во время свободного падения, называется конечной скоростью.

    2.2 — Силы
    • Объекты как точечные частицы

    Силы изменяют скорость или форму объектов.

    Единица силы — ньютон (Н).

    Объекты представлены в виде точечной массы, чтобы можно было представить силы в виде стрелок на диаграммах свободного тела.

    На схеме свободного тела силы, действующие на объект, представлены в виде стрелок, исходящих от точечной массы.

    Длина и направление стрелок соответствуют величине и направлению сил, действующих на исследуемое тело.

    Определение равнодействующей силы

    1. Разложите все действующие силы на горизонтальные и вертикальные составляющие
    2. Сложите горизонтальные компоненты
    3. Сложите вертикальные компоненты
    4. Объедините сумму горизонтальных компонентов и сумму вертикальных компонентов
    • Трансляционное равновесие

    Тело находится в поступательном равновесии, если результирующая сила, действующая на тело, равна нулю.Это означает, что тело либо находится в состоянии покоя, либо движется с постоянной скоростью. Например:

    • Масса в состоянии покоя
    • Лифт движется вверх с постоянной скоростью
    • Парашютист достигает предельной скорости
    • Законы движения Ньютона

    Первый закон Ньютона (Закон инерции) гласит, что тело остается в покое или движется с постоянной скоростью по прямой линии, если на него не действует внешняя сила. (Чистая сила = 0)

    Второй закон Ньютона гласит, что результирующая сила прямо пропорциональна ускорению и массе.(F = ma)

    Третий закон Ньютона гласит, что если тело A воздействует на тело B, то тело B прикладывает силу той же величины, но в направлении, противоположном телу A.

    Эта пара сил называется парой действие-противодействие, которая должна действовать на два разных тела.

    Трение — это неконсервативная сила, препятствующая движению. Если нет движения, то не будет силы, вызванной трением.

    Для двух твердых поверхностей, движущихся друг по другу, на трение будет влиять природа (шероховатость и т. Д.) Этих двух поверхностей.Однако площадь поверхности и скорость объекта не влияют на трение.

    Также существует два типа трения для твердых поверхностей: трение покоя и кинетическое трение. Статическое трение — это то, что останавливает движение объектов. Кинетическое трение — это то, что замедляет движущиеся объекты. Статическое трение всегда больше кинетического.

    Эти два типа трения определяются индивидуально своими константами µs и µk соответственно.

    Силы трения также зависят от нормальной силы, прилагаемой к поверхности, что приводит к Сила трения (статическая) = <мкс * Нормальная сила для неподвижных объектов и Сила трения (статическая) = мкс * Нормальная сила для объектов, которые движутся.

    2.3 — Работа, энергия и мощность

    Кинетическая энергия (KE) — это энергия тела за счет его движения и определяется уравнением

    • Гравитационная потенциальная энергия

    Гравитационная потенциальная энергия (GPE) объекта изменяется с его высотой и определяется уравнением

    Упругая энергия — это потенциальная энергия, запасенная в результате деформации упругого объекта, например, растяжения пружины, и определяется уравнением

    • Работа, выполняемая как передача энергии

    Выполненная работа измеряет передачу энергии за счет силы и является скалярной величиной.

    Работа, совершаемая W силой F над объектом, определяется уравнением

    На графике «сила-смещение» проделанная работа — это площадь под кривой.

    • Мощность как скорость передачи энергии

    Мощность (P) — это выполненная работа или выход энергии за время, определяемый уравнением:

    Для постоянной силы, действующей на объект с постоянной скоростью, мощность определяется уравнением: P = Fv.

    • Принцип сохранения энергии

    Энергия не может быть создана или уничтожена; его можно только изменить из одной формы в другую. Например:

    • Электрический нагреватель преобразует электрическую энергию в тепловую.
    • Падающий объект преобразует потенциальную энергию в кинетическую.

    Полная энергия изолированного тела остается постоянной. Другими словами, ΔKE + ΔPE = 0

    КПД — это отношение полезной выходной энергии к вложенной энергии в процентах, определяемое уравнением

    2.4 — Импульс и импульс
    • Второй закон Ньютона, выраженный скоростью изменения количества движения

    Импульс (p) задается уравнением

    Линейный импульс (p) — это вектор с тем же направлением, что и скорость объекта.

    Изменение количества движения объекта называется импульсом.

    Преобразование формулы, описывающей второй закон Ньютона, приводит к следующему выражению

    • Графики импульса и силы-времени

    Импульс задается областью графика «сила-время».

    • Сохранение количества движения

    Закон сохранения количества движения гласит, что сумма начального количества движения равна сумме конечного количества движения в замкнутой системе и может быть задана уравнением

    • Упругие соударения, неупругие соударения и взрывы

    Тип Общий импульс Полная кинетическая энергия y
    эластичный Консервированный Консервированный
    Неэластичный Консервированный Не сохраняется
    Взрыв Консервированный Не сохраняется

    Нравится:

    Нравится Загрузка…

    Что такое кинематика — Получите заметки, книги, формулы, уравнения по кинематике и физике

    Кинематика — одна из самых основных глав в механике, и вы также найдете много использования кинематики в нашей повседневной жизни после прочтения этой статьи. Большую часть нашей жизни мы ходили пешком, путешествуя отсюда туда, и поэтому мы также обеспокоены тем, сколько времени потребуется, чтобы переехать из одного места в другое. Кинематика занимается всеми этими процессами и упрощает нашу повседневную жизнь.

    В Индии большинство из нас фанаты крикета, и если вы когда-нибудь замечали, когда боулер подает мяч, на табло также отображается скорость мяча, а иногда повторы также показывают возможную траекторию мяча, которую необходимо определить для Leg Before Wicket ( LBW) тест. Это возможно, потому что кинематика помогает понять «движение» мяча. Движение — это не что иное, как смена позиции. В этой главе вы поймете некоторые основные термины, такие как расстояние, смещение, скорость, скорость, ускорение и т. Д.Вам следует прочитать эту главу очень внимательно, потому что вопросы из этой главы задаются не только на экзаменах 10 + 2, но и на различных других государственных экзаменах в разделе «Общие способности».

    Подготовка к работе в сети для JEE Main / NEET

    Crack JEE 2021 с программой онлайн-подготовки JEE / NEET

    Начать сейчас

    Использование DRS для тестирования LBW

    Кинематические темы

    Система отсчета, Движение по прямой — скорость и скорость, Равномерное и неравномерное движение, средняя скорость и мгновенная скорость, равномерно ускоренное движение, положение-время, графики скорости-времени, отношения для равноускоренного движения.Скаляры и векторы, нулевой вектор, скалярные и векторные произведения, единичный вектор, скорость, разрешение вектора, сложение и вычитание векторов. Движение в плоскости, относительная скорость, движение снаряда, равномерное круговое движение.

    Обзор кинематики

    В кинематике нас не интересует причина движения, нас интересует только движение объекта, простыми словами, нас не волнует, сколько импульса мячу дает котелок, нас интересует только движение шара после того, как он потерял контакт с рукой игрока.Движение объекта может быть одномерным, двухмерным или даже трехмерным.

    Движение объекта также может быть линейным или круговым. В этой главе вы познакомитесь с 3 уравнениями движения. Вы можете решить большинство вопросов кинематики, используя только эти три уравнения движения. Но прежде чем продолжить в этой главе, я предлагаю вам сначала развить некоторые базовые математические навыки, такие как дифференцирование, интегрирование, а также векторы. Я уверен, что вы изучали разницу между скалярными и векторными величинами в своих школьных учебниках.В этой главе вы познакомитесь с системой отсчета. Система отсчета — это система координат. Существует множество используемых систем координат, но наиболее распространена декартова система координат, состоящая из трех взаимно перпендикулярных осей. В действительности нет ничего стационарного, понятие стационарного используется с системой отсчета. Например, здание неподвижно, только если вы наблюдаете за ним на Земле, но если вы видите это здание с Луны, оно не неподвижно, поскольку Земля также вращается.Звучит интересно? Теперь вы можете разыграть своих одноклассников с помощью этой базовой системы координат.

    Вначале вы также часто будете делать ошибки или вычитать величины, например, если мяч брошен вертикально вниз из здания со скоростью «u», то какой будет конечная скорость по прошествии времени t? Будет ли оно равно «u + gt» или «u-gt»? Поэтому, чтобы свести к минимуму подобную путаницу, вы всегда должны использовать вектор в векторных величинах. Вопросы из этой главы сложны, когда они представлены в графической форме.Вы должны ознакомиться с основными уравнениями линии, параболы, чтобы получить решение.

    Предположим, ваш школьный автобус отправляется в 7:00 утра, тогда какое расстояние он проедет до 7:45?

    Решение:

    Дано

    Формулы кинематики

    Уравнения движения

    Как подготовить кинематику?

    Перед тем, как решать вопросы, вы должны хорошо владеть основами вычислений, вы должны уметь различать и интегрировать функции.Чтобы хорошо усвоить концепции, вы должны сначала решить вопрос из одномерного движения, а затем решить только двухмерное движение. Также изучите основы построения графиков. После прочтения темы пройдите несколько пробных тестов по этой конкретной теме, чтобы получить в ней силы. Иногда вопросы, связанные с относительным движением, движением снаряда и проблемой реки, являются сложными, поэтому больше практикуйтесь по этим темам.

    Примечания по кинематике
    • С помощью дифференцирования и интегрирования вы всегда можете найти функции x, v, a из одной заданной функции.Посмотрите на диаграмму ниже, чтобы понять это. Этот прием очень важен, потому что он будет использоваться во многих вопросах.

    • Самостоятельно составляйте графики для различных уравнений, с которыми вы сталкиваетесь при решении вопросов. Например. график x-t, график v-t, график a-t, график v-x, график a-x и т. д.

    • Используйте векторные обозначения в векторных величинах, направления очень важны, чтобы избежать глупых ошибок.

    • Не только запоминайте формулы, сначала проверьте их сами.

    • Для движения в 2-D или 3-D всегда разбивать (разделять) движение на составляющие по осям.

    • Существует общая подсказка для всех типов вопросов, в то время как практические вопросы всегда пишут, какие количества указаны в вопросе и что вам нужно найти.

    Книги по кинематике

    В книгах NCERT вы найдете вопросы уровня от простого до умеренного, сначала ответьте на них.Затем вы можете ответить на вопросы из «Понимания физики» Д.К. Панди или Х.К. Верма. Обе эти книги содержат вопросы хорошего качества. Но помимо этих книг вам следует также проводить пробные онлайн-тесты по темам. Наша платформа предоставит вам видео по всем темам, статьи на простом языке, чтобы облегчить обучение, и множество вопросов в пробных тестах.

    Примечания по физике для инженерных и медицинских экзаменов Кинематика

    — обзор | Темы ScienceDirect

    2.1 ОБЗОР

    Кинематика — это исследование движения системы тел без прямого учета сил или потенциальных полей, влияющих на движение. Другими словами, кинематика изучает, как импульс и энергия распределяются между взаимодействующими телами. Многие из взаимодействий, обсуждаемых в главе 1, представляют собой взаимодействия двух тел в том смысле, что в них участвуют две взаимодействующие сущности: снаряд по цели. Одним из важных следствий взаимодействия двух тел является то, что существует одна плоскость , которая содержит две взаимодействующие частицы.Таким образом, не будет компонента импульса вне плоскости взаимодействия. Следовательно, два тела, возникающие в результате взаимодействия, также должны находиться в одной плоскости. Следовательно, взаимодействия двух тел двумерны . Кроме того, имея всего два тела, можно легко определить центр масс , вокруг которого будут вращаться падающие и вылетающие частицы. Центр масс таков, что общий момент массы (масса X на расстоянии от центра масс) равен нулю.Следовательно, центр масс также таков, что полный импульс (масса × скорость) также равен нулю . Это значительно облегчает выполнение баланса импульса. Таким образом, кинематический анализ выполняется в центре масс. В разделе 2.2 мы обсуждаем взаимосвязь между системой отсчета центра масс и лабораторной системой отсчета (в которой записываются наблюдения). Помимо сохранения энергии и импульса, взаимодействия должны также подчиняться некоторым стехиометрическим и внутренним принципам сохранения, которые кратко изложены в разделе 2.4.

    При работе с частицами излучения малый размер частицы может привести к величине скоростей, близких к скорости света. Фотоны всегда движутся со скоростью света. Эти высокие скорости приводят к особым релятивистским эффектам, которые необходимо учитывать. Поэтому в разделе 2.3 рассматривается специальная теория относительности с особым вниманием к ее влиянию на сохранение энергии и импульса. Конечно, во многих случаях релятивистские эффекты не важны.Тем не менее, использование релятивистского кинематического анализа приведет к результатам, которые действительны как для высоких, так и для низких скоростей. По сути, классическая кинематика — это частный случай (приближение первого порядка) релятивистской механики, что демонстрируется некоторыми примерами в этой главе. Подробный математический анализ кинематики релятивистских частиц приведен в [18]. Релятивистская кинематика, часто называемая кинематикой Эйнштейна, обсуждается в разделе 2.5, а нерелятивистская классическая кинематика Ньютона рассматривается в разделе 2.6. В разделе 2.7 кинематический анализ применяется к тем взаимодействиям, которые обсуждались в главе 1, на которые не влияют электромагнитные поля.

    В большинстве случаев кинематика взаимодействия излучения касается взаимодействия одного снаряда с единственной мишенью, в отличие от взаимодействия молекул газа, где рассматривается коллективное движение газа. Газовая динамика требует общего анализа движения молекул газа, поскольку эти молекулы взаимодействуют друг с другом.В случае излучения плотность поля обычно довольно мала, и взаимодействие между частицами излучения маловероятно и просто игнорируется. Поэтому радиационные взаимодействия рассматриваются как процесс один-на-один (удар по цели), в котором падающее излучение передает энергию цели, не получая при этом энергии от нее. Однако, когда частицы излучения находятся в тепловом равновесии с атомами среды, частицы могут получать энергию от атомов мишени; процесс, который очень похож на газовый.Однако коллективный момент радиационных частиц рассматривается вероятностным образом, как обсуждалось в разделе 3.3.7 для теплового равновесия нейтронов.

    Как было показано в главе 1, существуют некоторые взаимодействия излучения, которые регулируются поведением электромагнитных волн, а не корпускулярной природой излучения. Кинематика этих волновых взаимодействий обсуждается в разделе 2.8.

    Уравнения движения | Квинтик Спортс

    4 квартал E Пример 13 — Уравнения движения

    Предлагаемое использование темы:
    Математика / физика (уровень A / AS), спортивные науки (степень 1/2)

    Введение

    Уравнения движения используются для описания различных компонентов движущегося объекта.Смещение, скорость, время и ускорение — кинематические переменные, которые могут быть получены из этих уравнений. Есть три уравнения, которые также называются законами постоянного ускорения, и поэтому могут применяться только тогда, когда ускорение постоянное и движение ограничено прямой линией. Три уравнения:

    1. v = u + при

    2. v² = u² + 2as

    3. с = ut + ½at²

    Где
    u = начальная скорость (мс‾¹)
    v = конечная скорость (мс‾¹)
    a = ускорение (мс‾²)
    t = время (с)
    с = смещение (м)

    Первое уравнение — это уравнение скорости-времени.Если ускорение постоянное, это означает, что скорость изменения скорости одинакова. Чем дольше происходит ускорение, тем больше изменяется скорость. Когда ускорение постоянно, скорость изменения скорости прямо пропорциональна времени. Если ускорение отсутствует, конечная скорость равна начальной скорости. Второе уравнение — это уравнение скорость-перемещение. Когда начальная скорость равна нулю, а ускорение постоянно, смещение прямо пропорционально квадрату скорости.Третье уравнение — это уравнение смещения-времени. Изменение смещения объекта прямо пропорционально квадрату времени, когда ускорение постоянное, а начальная скорость равна нулю.

    Снаряд имеет только две силы, действующие на него: силу тяжести и сопротивление воздуха. Сила тяжести влияет только на вертикальную скорость и имеет постоянное значение 9,81 мс². Сопротивление воздуха влияет на горизонтальную скорость, но обычно игнорируется, так как его влияние незначительно. Спроецированный объект, который выпущен и приземляется на одной высоте, имеет одинаковую начальную и конечную скорость.Кроме того, время, необходимое для достижения вершины, составляет половину общего времени движения, то есть время достижения максимальной высоты составляет ½t, когда начальная и конечная высота одинаковы. При попытке найти максимальную высоту объекта конечная скорость (v) принимается равной 0 мс‾¹. Это представляет вершину, поскольку скорость возвращается к нулю, когда снаряд меняет направление, чтобы вернуться вниз. Точно так же, если объект падает с высоты, начальная скорость (u) принимается равной 0 мс‾¹.

    Объективы

    1. Для расчета скорости, ускорения, времени и смещения подачи теннисного мяча и удара нетбола с использованием уравнений движения

    Методы

    • Видео были оцифрованы и откалиброваны с помощью программного обеспечения Quintic.
    • Данные были экспортированы в файл Excel, где они использовались для расчета линейного ускорения, горизонтального ускорения и вертикального ускорения. Графики были составлены с использованием этой информации.
    • Неподвижные изображения были сняты из видео, чтобы обозначить различные этапы учений

    Используемые функции программного обеспечения Quintic:

    • Модуль 1-точечной оцифровки
    • Калибровка
    • Интерактивный график и дисплеи данных
    • Экспорт данных
    • Захват нескольких изображений

    Результаты

    Два видео, посвященных теннису и нетболу, были проанализированы с использованием одноточечной оцифровки.Каждый кадр был оцифрован от выпуска мяча до конца видео.

    Два действия, которые были проанализированы, были подбрасыванием мяча при теннисной подаче и ударом от ворот при нетболе. В обоих этих навыках мяч проецируется вертикально, и когда он находится в воздухе, на него действует только сила тяжести.

    Теннисный стол

    Удар нетболом

    Начальная скорость (мс‾¹)

    5.30

    4,71

    Конечная скорость (мс‾¹)

    -1,37

    -2,75

    Расстояние до максимальной высоты (м)

    1,43

    1,13

    Время достижения максимальной высоты (с)

    0,54

    0.48

    Расстояние от максимальной высоты до конца (м)

    0,10

    0,38

    Время от максимальной высоты до конца (с)

    0,14

    0,28

    Общее расстояние (м)

    1,53

    1.52

    Общее время (с)

    0,68

    0,76

    Таблица 1: Результаты

    Графики 1 и 2 показывают скорость мячей при подбрасывании теннисного мяча во время подачи и броска нетбола соответственно, а графики 3 и 4 показывают расстояние, пройденное мячами. Выделенные области показывают изменение скорости и расстояния для соответствующих графиков.На картинках рядом с графиками показаны пути шаров для тех же рамок.

    Вертикальная скорость мяча на максимальной высоте в обоих навыках равна нулю. При расчете начальной скорости, времени или смещения этих действий от точки выброса до высоты пика конечная скорость равна нулю. Таким образом, при вычислении конечной скорости, времени или смещения от максимальной высоты до приземления начальная скорость равна нулю. Кроме того, когда объект проецируется, единственная сила, действующая на вертикальную скорость, — это сила тяжести (9.81 мс‾²), поэтому ускорение вертикальной составляющей снаряда всегда составляет 9,81 мс‾².

    График 1: Скорость тенниса

    Начальную скорость мяча во время подачи можно рассчитать, используя первое уравнение движения

    v = u + при

    График 1 показывает скорость теннисного мяча во время броска. Выделенная область показывает скорость мяча от точки выброса до высоты пика.Начальная скорость, когда мяч выпущен в кадре 35, составляет 5,30 мс², как рассчитано по уравнению. По мере увеличения высоты теннисного мяча скорость уменьшается из-за силы тяжести, действующей в противоположном направлении по отношению к теннисному мячу, заставляя его замедляться. В конечном итоге это приводит к тому, что скорость возвращается к нулю (кадр 62), что указывает на максимальную высоту теннисного мяча. Время, необходимое мячу для достижения максимальной высоты, составляет 0,54 с.

    График 2: Скорость нетбола

    Начальная скорость удара по воротам рассчитывалась также с использованием первого уравнения движения

    v = u + при

    График 2 показывает вертикальную скорость нетбола с выделенной областью, показывающей скорость от точки выброса до максимальной высоты.Как и в случае с теннисным броском, после того, как мяч выпущен, его скорость, которая изначально составляет 4,71 мс², постоянно уменьшается из-за силы тяжести, пока мяч не достигнет максимальной высоты и скорость не станет равной нулю. Время, необходимое мячу для достижения максимальной высоты, составляет 0,48 секунды.

    Смещение мяча от точки выпуска до максимальной высоты можно измерить с помощью второго или третьего уравнения движения, в зависимости от известных значений.

    График 3: Расстояние до теннисного мяча

    Смещение теннисного мяча рассчитывается с использованием второго уравнения движения.

    v² = u² + 2as

    График 3 показывает расстояние, пройденное мячом по вертикали. Пройденное расстояние рассчитывается только по кадру 35, поскольку это было, когда мяч был выпущен, и с этого момента на него действовала только сила тяжести. Начальная скорость была рассчитана ранее, а конечная скорость равна нулю. Ускорение составляет 9,81 мс², поэтому расчетное расстояние составляет 1,43 м.

    График 4: Расстояние нетбола

    Смещение нетбола было рассчитано с использованием третьего уравнения движения.

    с = ut + ½at²

    График 4 показывает расстояние до нетбола по вертикали. Расстояние рассчитывается между кадрами 55-79, так как это показывает расстояние от места выпуска мяча до достижения максимальной высоты. Начальная скорость была рассчитана по первому уравнению, ускорение — 9,81 мс², время — 0,48 секунды. Вертикальное расстояние до мяча составило 1,13 м

    Конечная скорость и расстояние от высоты пика до конца могут быть измерены с помощью тех же уравнений.При измерении от высоты пика до конца начальная скорость равна нулю.

    График 5: Расстояние и скорость от высоты пика до конца

    График 5 показывает расстояние и скорость мяча от максимальной высоты в кадре 62 до удара ракеткой в ​​кадре 69. Скорость мяча отрицательна из-за направления, в котором движется мяч. Кроме того, поскольку измеряется вертикальное расстояние, расстояние измеряется по отношению к высоте над землей.Таким образом, когда мяч меняет направление и начинает двигаться к земле, его высота уменьшается и, следовательно, расстояние уменьшается. Конечная скорость теннисного мяча по расчетам составляет -1,37 мс‾¹, а расстояние от максимальной высоты до конечной скорости составляет 0,10 м. Общее смещение теннисного мяча — это сумма расстояния, пройденного от точки выброса до высоты пика до удара ракеткой, и составляет 1,53 м.

    Рисунок 1: Теннисный сервиз

    На Рисунке 1 показаны различные этапы теннисной подачи.На первой картинке выпущен мяч; начальная скорость вычисляется в этой точке. На втором рисунке мяч находится на полпути к пику высоты. На третьем изображении мяч находится на максимальной высоте, скорость в этой точке равна нулю, а окончательное изображение — непосредственно перед ударом по мячу ракеткой, где рассчитывается окончательная скорость. Общее смещение — это расстояние, пройденное мячом от рисунка 1 до рисунка 4.

    График 6: Расстояние и скорость от высоты пика до конца

    График 6 показывает расстояние и скорость нетбола от максимальной высоты в кадре 79 до момента, когда мяч входит в ворота и не попадает в ворота в кадре 92.Как и в случае с подбрасыванием мяча в теннисе, расстояние уменьшается из-за того, что измеряется вертикальное расстояние. Кроме того, скорость увеличивается отрицательно из-за направления мяча. Конечная скорость нетбола находится с использованием первого уравнения движения. Расчетная конечная скорость составляет 2,75 мс‾¹. Смещение мяча за этот период определяется с помощью третьего уравнения движения и составляет 0,39 м. Общее смещение мяча при броске составляет 1,52 м.

    Рисунок 2: Удар нетбола

    На рис. 2 показаны различные этапы броска нетбола.На первой картинке выпущен мяч; начальная скорость вычисляется в этой точке. На втором рисунке мяч находится на полпути к пику высоты. На третьем изображении мяч находится на максимальной высоте, где скорость временно равна нулю, а на последнем изображении показано, как мяч входит в ворота, где рассчитывается окончательная скорость. Общее смещение — это расстояние, пройденное мячом от рисунка 1 до рисунка 4.

    Заключение

    Уравнения движения могут использоваться для расчета различных кинематических переменных, но только при постоянном ускорении и когда объект движется по прямой.Когда объект проецируется, сила тяжести является постоянным ускорением.
    Загрузки

    Физико-кинематика: объекты в движении: 11 класс

    Цель урока: Понимание значения кинематики, реальных примеров, элементов кинематики и введения в описание движения объектов с помощью чисел и уравнений.

    Научный стандарт нового поколения: Используйте математические представления явлений для описания объяснений.(HS-PS2-2), (HS-PS2-4)

    Материалы: Этот урок представляет собой вводный урок к более широкой концепции кинематики, больше похож на лекцию, чем на занятие. Заранее подготовьте проекции, в которых перечислены кинематические уравнения и пример. Вам также понадобится:

    Стартер

    Задайте вопрос: Кто-нибудь слышал раньше слово «кинематика»? (Позвольте учащимся ответить.) Если да, отлично. Вы можете мне сказать, что это значит? (Если кто-то ответит правильно, утвердите ответ и продолжайте.)

    Say: Кинематика происходит от греческого слова Kinesis, что означает движение. Обычно это определяется как изучение движения точек, объектов или групп объектов. В физике он принадлежит к разделу Механика.

    Кинематику не следует путать с кинетикой. Обе концепции являются ветвями движения и очень похожи. Однако кинетика — это изучение сил, которые влияют или вызывают движение (законы Ньютона), например, как трение влияет на ходьбу или бег или как фермент вызывает изменения в химической системе.

    Сегодня мы обсуждаем кинематику, науку о движущемся объекте или веществе без учета сил, которые его стимулируют. Хорошая идея — думать об этом как о том, как и почему. Кинетика учитывает, почему происходит движение, а кинематика занимается тем, как это происходит.

    Поскольку кинематика включает в себя значительный объем математики, мы должны сначала усвоить основные концепции и термины, чтобы лучше понять, почему мы выполняем математику, которая есть.

    Основная

    Say: Кинематика стреляет в баскетбольный мяч, вода стекает с водопада, движется поезд.Практически любой существующий объект имеет кинематику, пока он движется. Даже учебник на столе может быть примером кинематики, если учесть колебания атомов и молекул.

    Это краткая и краткая теоретическая часть кинематики, и в основном все, что к ней нужно. Однако мы можем перевести эти слова в математические выражения, называемые кинематическими уравнениями.

    Есть несколько элементов или величин, которые важны для математического расчета того, как происходит движение.Эти элементы являются векторными и скалярными величинами. Вы, наверное, уже слышали эти термины. Скалярные величины — это те, которые имеют только величину, то есть описываются только числовыми значениями. С другой стороны, векторные величины — это те, которые имеют как величину, так и направление.

    Скаляры, которые мы рассмотрим сегодня: (напишите на доске) :

    А векторов (напишите на доске):

    • рабочий объем
    • скорость
    • ускорение

    Позже вы увидите, что все вычисления будут сосредоточены на векторных величинах.

    Как именно работают кинематические уравнения? Возьмем, к примеру, скорость. Скорость — это скорость и направление движения. Итак, представьте, что я бросаю мяч в мусорное ведро, и он движется со скоростью около 1 км / ч. С того места, где я стою, он будет двигаться на восток в контейнер.

    Сделайте : бросьте мяч в пустой мусорный бак.

    Спросите : Скажем, скорость просто 1 км / ч на восток, верно? (Позвольте студентам ответить).

    Say: При расчете скорости движения кинематика учитывает расстояние, пройденное мячом, и время, необходимое для его остановки. Если бы мы могли это разбить, это означает, что мы рассматриваем начальную точку мяча (моя рука), конечную точку (мусорный бак) и его траекторию (путь, по которому он следует в пространстве). Вот почему физики часто называют кинематику «геометрией движения». По сути, кинематика изучает траектории точек, линий и других геометрических объектов при вычислении движения.

    Однако кинематика

    — это не только движение снаряда. Он также присутствует в вертикальном и горизонтальном движении.

    Do: Подбросьте мяч вверх и позвольте ему упасть на пол.

    Say: Кинематика здесь включает скорость мяча, когда он поднимается, ускорение (изменение скорости), когда он возвращается вниз, и время, необходимое для остановки от начала до конца движения.

    Do: Толкните игрушечную машинку по столу.

    Say: Здесь нас также интересует расстояние между точками A и B, скорость движения автомобиля и изменения скорости в пути.

    Следующим шагом для нас является собственно выполнение вычислений. К счастью, для этого используется набор основных формул.

    Do: Спроецируйте на экран следующую информацию. Дайте студентам время скопировать информацию.

    • v = v0 + при
    • d = ((v + v0) / 2) t
    • d = v0t + 1 / 2at2
    • v2 = v02 + 2ad

    Ключи

    • v0 = начальная скорость
    • v = конечная скорость
    • d = рабочий объем
    • a = ускорение
    • t = временной интервал

    Say: Эти формулы основаны на предположении, что ускорение является постоянным и что все параметры или величины относятся к одному и тому же направлению.

    Do: Спроецируйте следующий пример.

    Say: Теперь пример. Самолет движется по взлетно-посадочной полосе со скоростью 3,20 м / с2 в течение 32,8 с, пока наконец не оторвется от земли. Определите пройденное расстояние до взлета.

    Do: Работайте над решением вместе с учащимися.

    Параметры, которые у нас есть:

    • a = 3,20 м / с2
    • t = 32,8 с
    • v = 0 м / с (самолет на старте неподвижен)
    • d =?

    По формуле:

    Мы можем рассчитать расстояние (смещение) как:

    • d = (0 м / с) * (32.8 с) + 1/2 (3,20 м / с2) * (32,8 с) 2
    • d = 1721 м

    Say: Продолжая этот раздел, вы начнете понимать, насколько просто применить кинематику к кинетическим ситуациям.

    Обратная связь

    Спросите: Как, по вашему мнению, инженеры применяют кинематику к чему-то вроде самолета, автомобиля или другого оборудования? Как бы они использовали эти разные уравнения? (Разрешить учащимся ответить.)

    Say: Идите домой и бросьте какой-нибудь предмет через вашу гостиную — только ничего не ломайте.Попросите кого-нибудь использовать таймер с точностью до миллисекунд. Они должны запускать таймер, когда вы отпускаете мяч, и останавливать его, когда он ударяется о землю. Прежде чем бросить мяч, отметьте свое место на полу.

    Как только объект приземлится, измерьте расстояние между местом, где вы бросили мяч, и тем местом, где он приземлился. По имеющейся у вас информации вычислите, с какой скоростью мяч летел по воздуху. А теперь вычислите, как долго этот мяч должен лететь в воздухе, чтобы пролететь одну милю.

    Мы обсудим ваши расчеты на следующем занятии.

    Автор Энн Ифеани

    Участник образовательного мира

    Авторское право © 2021 Education World

    .

    Author: alexxlab

    Добавить комментарий

    Ваш адрес email не будет опубликован. Обязательные поля помечены *